Download as pdf or txt
Download as pdf or txt
You are on page 1of 157

PEDIA (JUNE)

1. Patients with this disorder are advised to avoid taking aspartame found in artificial sweeteners
a. Maple syrup urine disease
b. Galactosemia
c. Biotinidase deficiency
d. Phenylketonuria

2. Most common cause of choking in infants


a. Small objects
b. Liquids
c. Food
d. All of the above

3. The increase in plasma protein binding of drugs can increase their apparent volume of distribution
a. TRUE
b. FALSE

4. Which of the following is the most common chronic glomerular disease in children?
a. Alport syndrome
b. Membranoproliferative glomerulonephritis
c. Berger Nephropathy
d. Henoch-Schonlein Purpura Nephritis

5. The earliest and most reliable sign of shock a. Pallor


b. Poor pulses
c. Tachycardia
d. Decreased blood pressure

6. Which of the following intravenous solutions would be appropriate for a patient with severe hyponatremia secondary to
syndrome of inappropriate antidiuretic hormone secretion (SIADH)
a. Hypotonic solution
b. Hypertonic solution
c. Normotonic solution
d. Isotonic solution

7. Sacarlet Fever, an upper respiratory tract infection associated with a characteristic rash is caused by what
microorganism?
a. Staphylococcus aureus
b. Coagulase-negative Staphylococcus
c. Group A Streptococcus
d. Streptococcus pneumoniae

8. Clinical manifestation of Marasmus EXCEPT:


a. Poor appetite
b. Subnormal body temperature
c. Bradycardia
d. Mental changes

9. What is the most frequent cause of common colds in both adults and children?
a. Corona virus
b. Adenovirus
c. Rhinovirus
d. Respiratory syncytial virus
10. 15-month old girl presented in ER with history of URI, and scattered petechiae and ecchymoses over the body and
lower extremities. Physical examination normal and no hepatosplenomegaly. WBC 14,000, Hb 12.8, Plts 5,000.
Diff count: normal. Next step:
a. Perform bone marrow examination
b. Non-accidental injury, request for skeletal survey to rule out bony fractures
c. Treat with IVIG
d. Administer platelet transfusion

11. True about type 2 diabetes mellitus EXCEPT:


a. Children with this type of diabetes are usually obese
b. Acanthosis nigricans is present in the present in the majority of patients
c. Patients usually have normal triglyceride and cholesterol levels at diagnosis
d. Characterized by peripheral insulin resistance

12. Which of the following is the most important physical assessment parameter that you would consider when assessing
fluid and electrolyte imbalance?
a. Osmotic Pressure
b. Intake and output
c. Skin Turgor
d. Cardiac Rate and Rhythm

13. A 3-year-old boy presents with failure to thrive and is found to have Congenital Hypoplastic Anemia (Diamond
Blackfan Anemia) Syndrome. The clinical features may be associated with this syndrome EXCEPT:
a. Profound anemia by 2-6 months of age
b. Normochromic, macrocytic, reticulopenia and insufficient or absent RBC precursors in otherwise normally cellular
bone marrow
c. 50% have extrahematopoietic anomalies
d. Autosomal recessive inheritance

14. A 3-year-old boy presents with failure to thrive and is found to have Congenital Hypoplastic Anemia (Diamond
Blackfan Anemia) Syndrome. The clinical features may be associated with this syndrome EXCEPT:
a. Profound anemia by 2-6 months of age
b. Normochromic, macrocytic, reticulopenia and insufficient or absent RBC precursors in otherwise normally cellular
bone marrow
c. 50% have extrahematopoietic anomalies
d. Autosomal recessive inheritance

15. Microcytic anemia is a characteristic laboratory abnormality of all listed disease EXCEPT:
a. Iron deficiency anemia
b. Lead poisoning
c. Sickle cell disease
d. Thalassemia trait

16. Working memory is managed by:


a. Brain stem
b. Cerebellum
c. Frontal area
d. Temporo-parietal lobe

17. Cardinal signs of intestinal obstruction


a. Vomiting
b. Abdominal Distension
c. Constipation/Obstipation
d. a & c only
e. all of the above
18. True about diabetic ketoacidosis EXCEPT:
a. It is the end result of the metabolic abnormalities resulting from a severe deficiency of insulin or insulin effectiveness
b. Reversal of DKA is associated with inherent risks that include hypoglycaemia, hypokalemia and cerebral edema
c. Insulin is given at beginning of therapy to accelerate movement of glucose into cell
d. DKA results in severe dehydration and patients must be given oral rehydrating solutions together with
intravenous hydration at the start of treatment

19. PA is a 10 y/o female brought to your clinic because of recurrent coughing episodes. Her mother said that the cough
has been on-and-off since patient was 3 y/o and she was always diagnosed as having an upper respiratory tract infection.
You elicited from the history that the patient also had atopic dermatitis at 1 month old, and she was shifted from formula
milk to breast milk later on because of confirmed cow’s milk allergy through serum specific IgE testing. Which type of
childhood asthma pattern does this patient have?
a. Transient early wheezing
b. Persistent atopy-associated asthma
c. Nonatopic wheezing
d. Asthma with declining lung function

20. ASO titer is elevated in post Streptococcal AGN secondary to pharyngitis; the same is true with ASO titer in PSAGN
secondary to dermatoses
a. True
b. False

21. True of the mechanism of vaccines, EXCEPT:


a. Stimulation of a person’s immune system through the administration of antigens
b. Administration of preformed human antibodies to person before or after exposure to infectious agent
c. None of the above
d. All of the above

22. Prolactin receptors are established within the


a. 8 days after delivery
b. 3 days after delivery
c. 5 days after delivery
d. 10 days after delivery
23. True about the diagnosis of congenital hypothyroidism EXCEPT:
a. Infants are identified by newborn screening programs
b. Serum levels of T4 of free T4 are elevated
c. Retardation of osseous development can be shown radiographically at birth and indicates some deprivation of thyroid
hormone during intrauterine life
d. Demonstration of ectopic thyroid tissue in scintigraphy is diagnostic of thyroid dysgenesis

24. Subacute Panencephalitis is a chronic complication of what viral infection?


a. Rubella
b. Measles
c. Mumps
d. Varicella

25. Endogenous molecules produced by or released from damaged and dying cells
a. DAMPS
b. PAMPS
c. PRRs
d. Soluble recognition molecules

26. Cirrhosis and congestive heart failure are associated with decreased hepatic blood flow and hence decrease the
clearance of drugs with high extraction ratios
a. TRUE
b. FALSE
27. Clinical manifestations of Graves’ disease EXCEPT:
a. Symptoms develop gradually
b. Earliest signs in children may be emotional disturbances accompanied by motor hyperactivity
c. Nodular goiter is found in almost all patients
d. Skin is smooth and flushed, with excessive sweating

28. Newborn screening is ideally done _____ after birth


a. 24 hours
b. 48 hours
c. 5 days
d. days

29. Protein of Human Mature Milk


a. 1.1%
b. 3%
c. 2%
d. 7%

30. True about type 1 diabetes mellitus EXCEPT:


a. Onset occurs predominantly in childhood, with a median age of 7-15 yrs
b. Patients are not insulin-dependent and infrequently develop ketosis
c. Characterized by autoimmune destruction of pancreatic islet β cells
d. Most important clue in diagnosis is an inappropriate polyuria in any chils with dehydration, poor weight gain, or “the
flu”

31. Signs of increased respiratory effort include the following, EXCEPT:


a. Nasal flaring
b. Abdominal bloating
c. Chest retractions
d. Head bobbing

32. The expanded newborn screening will increase the panel of disorders from 6 to ___
a. 20
b. 28
c. 32
d. 40

33. Child rolls over, laughs out loud, says mama, grasp objects with thumb and forefinger
a. 6 mos
b. 8 mos
c. 10 mos
d. 12 mos

34. Criteria for malnutrition using Wellcome classification EXCEPT:


a. Weight loss in terms of weight for age
b. Presence or absence of edema
c. Weight loss in terms of height for age
d. All of the above

35. Occasional signs of Kwashiorkor


a. Diffuse pigmentation of skin
b. Moon face
c. Anemia
d. Flaky paint rash
36. Most children begin to walk independently at around what month of age?
a. 10-12 mos
b. 11-13 mos
c. 12-15 mos
d. 11-14 mos

37. A 10-year old female at OPD was brought at OPD by her mother due to uncontrollable “jerking” movement of right
arm lasting a minute. She happens to be aware during the event. Your impression is a seizure disorder of what type?
a. Tonic
b. Clonic
c. Myoclonic
d. Focal

38. True of achalasia


a. Peristalsis normal in the body of esophagus
b. Abundant Argyrophil ganglion cells
c. Failure of LES to relax in response to swallowing
d. a & c only
e. all of the above

39. Which statement/s is/are TRUE?


a. Teratology deals not only with congenital malformations observed grossly at birth, but also embraces any adverse
effects, morphological, biochemical, behavioural, etc., induced during fetal life and detected at birth or later
b. Dysmorphism is a broad term that includes both functional and structural abnormalities of the fetus
c. BOTH
d. NEITHER

40. In the Philippines, this disorder has the highest incidence of confirmed cases (as of Dec 2016)
a. Congenital hypothyroidism
b. Galactosemia
c. G6PD deficiency
d. Congenital adrenal hyperplasia

41. In girls, UTI in 75-90% of all infection is caused by what bacteria?


a. E. coli
b. Klebsiella spp
c. Proteus spp
d. Enterococcus

42. What is the most common cause of bacterial meningitis in children older than 1 month of age in the US?
a. Streptococcus pneumonia
b. Neisseria meningitidis
c. Haemophilus influenza type B
d. A & B
e. All of the above

43. Which body compartment contains the greatest relative amount of water?
a. Extracellular
b. Intracellular
c. Plasma
d. Transcellular

44. The antibody involved in allergic reactions


a. IgA
b. IgM
c. IgG
d. IgE

45. Object permanence is achieved by what month of age?


a. 6 mos
b. 7 mos
c. 8 mos
d. 9 mos

46. What triggers signals the brain to increase the output of ADH for water conservation?
a. Thickened saliva signals the sympathetic nervous system osmoreceptors and the hypothalamus detect the increase
b. The osmotic pressure of the body signals the posterior pituitary to release ADH chemoreceptors and the renal
tubule sense the increase
c. Viscosity of renal filtrate signals the hypothalamus which, in turn, signals the posterior pituitary and the
juxtaglomerular apparatus senses the greater
d. Osmotic pressure in the blood triggers the release of ADH

47. Increased skin hydration and an underdeveloped epidermal barriers (stratum corneum) are responsible for the
heightened percutaneous absorption among neonates
a. TRUE
b. FALSE

48. These are modified viruses or bacteria that are weakened but retain the ability to replicate and produce immunity
without causing illness
a. Inactivated vaccines
b. Live attenuated
c. None of the above
d. All of the above

49. True about the diagnosis of primary immunodeficiencies EXCEPT:


a. The most frequent symptom prompting the diagnosis is the presence of recurrent or unusually severe infections
b. Viral infections, recurrent Candida infections and opportunistic infections are generally suggestive of a detective
antibody response
c. The performance of laboratory tests should be guided to some extent by the clinical findings
d. Skin infections and deep seated abscesses primarily reflect innate immune defects

50. The most serious consequence of potassium imbalance is __________


a. Seizure
b. Nerve Damage
c. Renal Failure
d. Cardiac Abnormalities

51. During neurological evaluation, the level of alertness of the newborn is dependent on EXCEPT:
a. Interaction with the environment
b. Room Temperature
c. Age of Gestation
d. Time of last feeding

52. Factors affecting accessibility of the drug to the fetus include:


a. Rate of delivery of the drug to the placenta
b. Concentration of the drug in maternal arterial blood and uterine blood flow
c. Structure and biochemical properties of the placenta
d. All of the above
e. None of the above
53. A 3-yeal old male was brought to the ED due to seizure that characterized repeated where in the patient does not
regain consciousness in between attacks. What type of seizure does your patient has?
a. Generalized seizure
b. Partial seizure
c. Unclassified seizure
d. Status epilipticus

54. The predominant type of protein in human milk


a. Whey
b. Casein
c. Both
d. Neither

55. Hirschprung’s disease- highly suspected in: a. Failure to pass meconium during 1st 40 hours of life
b. Progressive abdominal distention
c. Infant refuses feeding
d. Billous vomitus
e. All of the above

56. Understanding of variability in drug disposition is essential if children are to receive rational and appropriate drug
therapy
a. TRUE
b. FALSE

57. Hutchinson teeth is associated with what type of infection?


a. Chlamydia trachomatis
b. Treponema pallidum
c. Mycoplasma genitalium
d. Rickettsia ricketsii

58. Tantrum peaks in prevalence between what age?


a.2-3 yr of age
b.3-4 yr of age
c.2-4 yr of age
d.3-5 yr of age

59. A microscopic ball of blood vessels that serves as an ultra-filter that allows passage of most dissolved substances in
plasma while keeping blood cells, plasma proteins and lipids inside capillary vessels
a. Nephron
b. Glomerular
c. Both
d. Neither

60. Pyloric Stenosis:


a. Usually occurs in 2-3 weeks old premature female infant
b. Individual has voracious appetite, yet fails to thrive
c. (+) pyloric murmur
d. b & c are correct
e. all of the above are correct

61. Bed-wetting is normal up to what age in boys?


a. 4 yrs old
b. 5 yrs old
c. 6 yrs old
d. 7 yrs old
62. Runs well, knows age and sex, pretends in play, copies circle
a. 24 mos
b. 30 mos
c. 36 mos
d. 48 mos

63. Ranks as the no. 1 causative organism in children during the first episode of UTI and reccurence
a. Pseudomonas
b. E. coli
c. Both
d. Neither

64. Visual acuity reaches 20/20 by what age?


a. 4 yrs old
b. 5 yrs old
c. 6 yrs old
d. 7 yrs old

65. What is the best known clinical syndrome caused by Epstein - Barr virus?
a. Infectious mononucleosis
b. Erythema infectiosum
c. Herpangina
d. Roseola infantum

66. Contraindications to breastfeeding EXCEPT: a. HIV


b. Tuberculosis
c. Herpes Lesions on mother’s breast
d. A & B
e. A,B,C

67. What is the most common form of acute upper respiratory obstruction?
a. Croup
b. Acute Epiglottitis
c. Bacterial tracheitis
d. Acute Infectious Laryngitis

68. Metabolites of Chloramphenicol and Aminoglycosides accumulate due to the immature function of the liver including
the less developed hydroxylation pathway which is supposed to convert chloramphenicol to the highly water-soluble
moiety.
a. TRUE
b. FALSE

69. 3-year old patient is brought to ER with complaints of feeling tired over the past 3 days. Patient is pale, jaundiced,
with the spleen tip palpable. CBC: Hgb 5, Retic count 5%, LDH increased. What does this sound like?
a. Hemolysis
b. Thalassemia
c. IDA
d. Aplastic anemia

70. Milk Code of the Philippines is under


a. E.O No. 5
b. E.O No.15
c. E.O No.51
d. E.O No.115
71. The most common hemoglobinopathy variant in Southeast Asia
a. Hemoglobin C
b. Hemoglobin D
c. Hemoglobin E
d. Hemoglobin S

72. True of Passive immunization, EXCEPT:


a. Transplacental transfer of maternal antibodies
b. Administration of an antibody produced by a person or animal to another person and provides temporary protection
against some infection
c. Vaccines contain 1 or more antigen that interact with the immune system
d. These include blood products used for transfusion

73. Policies that promote breastfeeding in Maternal Ward


a. Give Prelacteals
b. Scheduled feeding
c. Demand feeding
d. Feeding should start 2 hours after delivery

74. A child vocabulary balloons from 10-15 words at 18 mos to between 50 and 100 at what age?
a. 20 mos
b. 24 mos
c. 30 mos
d. 46 mos

75. Milk production and secretion are responsive to:


a. Suckling Reflex
b. Oxytocin
c. Both
d. Neither

76. Gastroesophageal reflux (GERD)


a. Lower esophageal sphincter (LES) competent, high LES pressure
b. Competent LES facilitates reflux of acid contents into the esophagus
c. Mainly self-limited
d. Poor intake from the start
e. Surgery as initial mode of management

77. The following are inactivated virus vaccine, EXCEPT:


a. IPV
b. Rabies
c. OPV
d. Hepatitis A vaccine

78. Membrane attack complex


a. C3,4,5,6,7
b. C1,2,3,4,5
c. C5,6,7,8,9
d. C2,3,4,5,6

79. The major hormones affecting breast development


a. Progesterone
b. Estrogen
c. Both
d. Neither
80. The single most common finding among infants with congenital rubella syndrome?
a. Encephalitis
b. Transient Aplastic Crisis
c. Cerebellar Ataxia
d. Nerve Deafness

81. The following pharmacologic agents are used in the treatment of asthma EXCEPT:
a. Anticholinergic agents
b. Alpha-adrenergic agents
c. Leukotriene-modifying agents
d. Glucocorticoids

82. Manifestations of increased intracranial pressure in infants, EXCEPT:


a. Setting sun sign
b. Bulging fontanelle
c. Large head
d. Headache

83. Most common position used by mothers to breastfeed


a. Cradle Hold
b. Foot Ball Hold
c. Side Lying Position
d. A & B
e. A,B,C

84. The average child points to major body parts by what age?
a. 12 mos
b. 15 mos
c. 18 mos
d. 20 mos

85. In neonatal circulation, the ff is TRUE


a. Heart murmurs are caused by turbulent blood flow
b. In the presence of cardiopulmonary disease continued patency of ductus arteriosus may allow left bidirectional shunting
c. Both
d. Neither

86. The following statement/s is/are TRUE


a. Drug metabolism or biopharmaceutics is the chemical transformation of a drug by the biological environment from
metabolites
b. Pharmacokinetics refers to what the drug does to the body after administration
c. Pharmacodynamics refer to what happens to the drug prescribed and administered to the patients d. All of the above
e. None of the above

87. Jake was playing when he suddenly stopped, appeared dazed and pale. He seemed unaware that you were calling his
name. He fidgets with his shirt and shit’s buttons then goes around in circles for about 20 seconds. Classify this patient’s
seizure type.
a. Tonic-clonic
b. Clonic
c. Absence
d. Atonic
88. Waterlow classification:
a. Wasting is indicative of acute malnutrition
b. Stunting is indicative of chronic malnutrition
c. Both
d. Neither

89. Significant bacteuria occurs when there’s about 10 to the 3rd power colony forming units\mL of urine in a freshly
voided clean catch urine or any bacterial growth in the urine collected by a suprapubic aspiration.
a. TRUE
b. FALSE

90. The following are true of factors that influence the immune response to vaccination, EXCEPT:
a. Nutritional status
b. Route of Administration
c. All of the above
d. None of the above

91. A 2-year old boy presents for evaluation of chronic pruritic eruption. His medical history is remarkable for recurrent
epistaxis, otitis media, and pneumonia. Physical examination reveals erythematous, slightly scaling patches on the trunk
and in the antecubital and popliteal fossa. Petechiae are present profusely. This is most suggestive of:
a. Ataxia telangiectasia
b. Atopic dermatitis
c. Langerhans cell histiocytosis
d. Wiskott-Aldrich syndrome

92. Which is TRUE about atopic dermatitis (AD)?


a. Acute atopic dermatitis lesions consist of erythematous, excoriated and scaling papules
b. Intense pruritus and a personal/family history of atopy are the cardinal features of AD
c. Older children have lesions involving the extensors of the extremities
d. The diaper area is spared in infants with atopic dermatitis

93. CJ, a 10-y/o male, was brought to your clinic because of recurrent colds. He has been having daily symptoms of runny
nose, sneezing, itchy nose and eyes for 2 months already. When asked about his academic performance, the mother says
that he does well in school and participates in school activities. He also sleeps soundly, and doesn’t wake up in the middle
of the night because of his symptoms. No previous consults done, and the mother only self-medicated with a nasal
decongestant and as needed, antihistamine. How will you classify this patient?
a. Mild intermittent allergic rhinitis
b. Moderate-severe intermittent allergic rhinitis
c. Mild persistent allergic rhinitis
d. Moderate-severe persistent allergic rhinitis

94. Nephrotic syndrome is 90% idiopathic with mesangial proliferative type as the most common histological type
comprising 85%
a. TRUE
b. FALSE

95. The Glasgow Coma Scale (GCS) is scored based on response to all of the following, EXCEPT:
a. Eye opening
b. Verbal response
c. Motor response
d. Cardiac output

96. In PEM iceberg prevalence


a. Mild PEM 15-40%
b. Moderate PEM 10-25%
c. Severe PEM 10-15%
d. No evidence of PEM >50%

97. Airway obstruction in the pediatric age group is treated with a sequential approach that includes the following,
EXCEPT:
a. Head-tilt/chin-lift maneuver
b. Inspection for a foreign body
c. Finger-sweep clearance or suctioning of a visualized foreign body
d. Blind suctioning or finger sweeps

98. Infants who are breastfed and have CMPA should do the following, EXCEPT:
a. Stop breastfeeding
b. Mother should restrict the amount of dairy products from her diet
c. Switch to soy-based formula
d. Switch to hypoallergenic formula.

99. Binding of complement protein to an antibody attached to a microbe


a. Alternative pathway
b. Classical pathway
c. Lectin pathway
d. Membrane attack complex

100. Handedness is usually established by what age?


a. 3 yrs old
b. 4 yrs old
c. 5 yrs old
d. 6 yrs old
PEDIA (DECEMBER)

1.
a. Compressed History
b. Problem Oriented Interview
c. Both
d. Neither

2. The scope of health history would include EXCEPT:


a. Patient’s concern
b. Physical Examination
c. Goals of Assessment

3. A chief complaint maybe recorded as


a. My head aches
b. I came for my annual check up
c. Both
d. Neither

4. Pediatric feeding history would include:


a. Breastmilk or cow’s milk
b. Solid Food
c. Flouride use
d. A & B only
e. A, B, C

5. Normal lung variation in Pediatrics Age Group


a. Softer auscultated breath sounds
b. Use of abdominal muscle in the neonate during respiration is a sign of a lung problem
c. Both
d. Neither

6. The following is a normal age-related variation of the eyes of an infant


a. Pseudostrabismus
b. Focusing with both eyes by one month
c. Gazing at and following bright objects at 6 months
d. All of the above

7. Respiratory rate of 30-50 cycles per minute


a. Premature infants
b. Newborns
c. Toddlers
d. None of the above

8. Normal systolic blood pressure of infants 1/1


a. 50-70 mmHg
b. 70-100 mmHg
c. 80-100 mmHg
d. None of the above

9. Newborns may lose how many percent of their birth weight in the first 3-4 days?
a. 5%
b. 10%
c. 15%
d. 20%
Other:
10. Posterior fontanels close by
a. 2-5 months
b. 12-18 months
c. 6-9 months
d. 1-2 months

11. The child has greater energy needs because:


a. Smaller body surface area
b. Greater activity
c. Both
d. Neither

12. Basal metabolism means:


a. Amount of heat given off or spent during activity
b. Adult value is 50-55 kcal/day
c. Both
d. Neither

13. Function of Carbohydrates:


a. Antiketogenic
b. Temperature regulation
c. Both
d. Neither
Other:

14. Protein deficiency results in:


a. Negative Nitrogen Balance
b. Marasmus
c. Both
d. Neitherption

15. Potassium
a. Needed for muscle contraction
b. Deficiency would result to heart block
c. Both
d. Neither

16. Policies that promote breastfeeding in maternal ward


a. Feeding 2 hours after delivery
b. Scheduled feeding
c. Both
d. Neither

17. Colostrums
a. More on fat and sugar
b. Low specific gravity
c. Both
d. Neither

18. Weaning:
a. Transitional period between 6 months to 2 years
b. During illness foods should be withheld due to impaired digestion
c. Both
d. Neither
19. Milk lobes directly open to:
a. Lactiferous sinuses
b. Milk ducts
c. Both
d. Neither

20. Sucking of sensitive fibers of the nipple would send first impulse to the:
a. Hypothalamus
b. Pituitary gland
c. Both
d. Neither

21. Function of Vitamin B6 would include


a. Involved in the catabolism of amino acid
b. Involved in the synthesis of amino acid
c. Involved in the synthesis of Norepinephrine
d. B & C only
e. A, B, C

22. Etiology of Vitamin B3 Deficiency


a. Diet low in Niacin
b. Diet low in Tryptophan
c. Excessive Corn Consumption
d. B & C only
e. A, B, C

23. Indication of chronic folate deficiency


a. RBC Folate level
b. Serum Iron and Vitamin B12 are decreased
c. Anemia microcytic
d. Serum Folic Acid is decreasing

24. Etiology of Vitamin B12 Deficiency EXCEPT:


a. Strict Vegetarian Diet
b. Commonly seen in Kwashiorkor or Marasmus
c. Consumption of the B12 intrinsic factor complex
d. Congenital Pernicious Anemia

25. Clinical manifestation of Vitamin B12 deficiency EXCEPT:


a. Megaloblastic Anemia
b. Neurological manifestations
c. Tongue is rough and red
d. Tongue is smooth and painful

26. Reliable and sensitive index of Vitamin B12 Deficiency


a. Serum LDH activity is decreased
b. Urinary excess of methylmalonic acid
c. Both
d. Neither

27. Functions of Ascorbic Acid


a. Collagen, a major connective tissue, is formed from choline requiring Ascorbic Acid
b. Ascorbic Acid is invlved in the oxidation reaction in the synthesis of steroids and epinephrine
c. Both
d. Neither
28. The following are endocrine causes of obesity EXCEPT:
a. Hypothyroidism
b. GH deficiency
c. Hypoparathyroidism
d. Cushing’s Syndrome

29. What type of Vitamin Deficiency is Pellagra?


a. Vitamin B2
b. Vitamin B3
c. Vitamin B6
d. Vitamin B12

30. The presence of horizontal depression along the lower anterior chest knowns as Harrison’s groove indicates what type
of Vitamin Deficiency?
a. Biotin Deficiency
b. Pyridoxine Deficiency
c. Vitamin C Deficiency
d. Vitamin D Deficiency

31. Late Vitamin K deficiency bleeding most commonly occurs at what week of age?
a. 2-12 weeks
b. 4-10 weeks
c. 4-12 weeks
d. 2-10 weeks

32. The first clinical manifestation of Hypovitaminosis A


a. Bitot’s spots
b. Xerosis Conjunctiva
c. Nyctalopia
d. Photophobia

33. Manifestations of Tocopherol Deficiency EXCEPT:


a. Risk of retrolental fibroplasia in premature
b. Focal Necrosis of stricted muscle
c. Decrease platelet adhesions
d. Anemia in kwashiorkor

34. Etiology of Tocopherol Deficiency


a. Cystic Fibrosis due to malabsorptive state
b. Decrease in Iron administration
c. Both
d. Neither

35. Active form of Vitamin A would include:


a. Retinol
b. Retinaldehyde
c. Retinoic Acid
d. A & C
e. A, B, C

36. Pneumonia is infection of the lungs caused by, EXCEPT:


a. Virus
b. Bacteria
c. Fungie
d. Neither
37. True of cough in patient suffers from pneumonia
a. Cough is usually dry
b. Sputum is even blood stained
c. B and C
d. Cough is chronic

38. True of vaccines as prevention in pneumonia


a. Vaccines do not help populations predisposed to pneumonia
b. Vaccines include Pneumococca and influenza
c. A and B
d. Neither

39. Pneumonia that is acquired in a hospital environment


a. Idiopathic Pneumonia
b. Community Acquired Pneumonia
c. Nosocomial Pneumonia
d. A and C

40. True of hospital acquired pneumonia


a. Less serious than Community Acquired Pneumonia
b. Serious than Community Acquired Pneumonia due to existence of other illness
c. Complications of hospital acquired pneumonia can be contributed by the resistant form of hospital organisms
d. B and C

41. What causes Atypical Pneumonia?


a. Mycoplasma
b. RSV
c. Influenza
d. Pneumococcus

42. True of bacterial Pneumonia


a. Results in complications like sepsis
b. Results in Lung abscess
c. Results in Pleutal Effusion
d. All of the above

43. The flue vaccine is given every:


a. 5 years
b. 2 years
c. 1 year
d. 10 years

44. Atypical Pneumonia can be caused by the following EXCEPT:


a. Klebsiella
b. Mycoplasma
c. Legionella
d. Corona Virus

45. Which of the following best describes Pleural Effusion?


a. The collapse of alveoli
b. The fluid in the alveolar space
c. The accumulation of fluid between the linings of the pleural space
d. The collapse of bronchioles

46. The cyanosis that accompanies bacterial pneumonia is primarily caused by which of the following?
a. Pleural Effusion
b. Decreased oxygenation of the blood
c. Decreased cardiac output
d. Inadequate peripheral circulation

47. Which of the following antituberculous drugs can cause damage to the eight cranial nerve?
a. Isoniazid
b. Ethambutol hydrochloride
c. Streptomycin
d. Ciprofloxacin

48. Which of the following diagnostic tests is defiitive for TB?


a. Chest X-ray
b. Tuberculin test
c. Mantoux test
d. Sputum culture

49. A client with shortness of breath has decreased to absent breath sounds on the right side, from the apex to the base.
Which of the following conditions would best explian this?
a. Acute Asthma
b. Chronic Bronchitis
c. Spontaneous Pneumothorax
d. Pneumonia

50. A client’s ABG analysis reveals a pH of 7.18, PaCO2 of 72mmHg, PaO2 of 77 mmHg, and HCO3 of 24 mEq/L. What
do these values indicate?
a. Respiratory Acidosis
b. Respiratory Alkalosis
c. Metabolic Acidosis
d. Metabolic Alkalosis

51. A client has active TB. Which of the following symptoms will he exhibit?
a. Chest and lower back pain
b. Headache and Photophobia
c. Chills, fever, night sweats, hemoptysis

52. A client who is HIV+ has had a 7mm PPD skin test.
a. Equivocal results
b. Negative
c. Positive
d. Repeat testing

53. A patient who is experiencing post streptococcal glomerulonephritis has edema mainly in the face and around the
eyes. Edema is more prominent at;
a. Bedtime
b. Afternoon
c. Morning
d. Lunch

54. Which of the following is not a sign and symptom of acute glomerulonephritis (poststreptococcal)
a. Hypertension
b. Massive proteinuria
c. Facial edema
d. None of the above

55. Post streptococcal glomerulonephritis is a type of Nephrotic syndrome, which means there is a leakage of both red
blood cells and protein from the inflamed glomerulus into the filtrate.
a. True
b. False
c. Neither

56. Acute post streptococcal glomerulonephritis is primarily caused by


a. Accumulation of antiglomerular basement membrane antibodies.
b. Antigen-antibody complex deposition in the glomerular capillaries and inflammatory damage
c. Inflammatory factors that stimulate cellular proliferation of epithelial cells
d. Swelling of mesangial cells in the Bowman space in response to the presence of bacteria

57. What is the most common cause of acute glomerulonephritis?


a. Fungal infections
b. Staph infection on the skin
c. Strep Infection in the throat
d. A and B

58. At what gestational week will surfactant appear?


a. 17-26 weeks
b. 6 weeks
c. 120 weeks
d. 36 weeks

59. What are surfactant molecules exerted and made of?


a. Alveolar cell type 1
b. Alveolar cell type 2
c. Glomerular cells
d. Langerhans cells

60. What is the primary cause of respiratory distress?


a. Lung hyperplasia
b. Lung hypoplasia
c. Lung agenesis
d. Dextrocardia

61. What does surfactant do except?


a. It decreases the alveolar surface tension
b. It increases the alveolar surface tension
c. It increases the lung compliance
d. Decreases the work of breathing

62. What are some other risk factors for Neonatal Respiratory Distress Syndrome?
a. Maternal Diabetes
b. Normal Delivery
c. Chronic Hypertension
d. A and C

63. What are the signs of RDS except?


a. Intercostal recession
b. Cyanosis
c. Vomiting
d. Respiratory distress shortly after birth

64. What would you see on a CXR in RDS except?


a. Air bronchograms and diffuse granular patterns
b. Consolidation
c. Ground glass appearance
d. A and C
65. What is the most common acyanotic congenital heart disease?
a. PDA
b. VSD
c. ASD
d. TGA

66. Boot shape x-ray is seen in?


a. TAPVR
b. TGA
c. TOF
d. TVA

67. Femoral-radial delay is seen in?


a. Coarctation of aorta
b. Interrupted aortic arch
c. Ventricular Septal defect
d. Patent ductus arteriosus

68. In the fetal circulation, what receives the largest amount of combined ventricular output and has the lowest vascular
resistance?
a. Superior Vena Cava
b. Placenta
c. Lungs
d. Heart

69. Snowman’s sign on chest x-ray


a. TAPVR
b. PAPVR
c. ASD
d. VSD

70. Most common cyanotic heart disease after first week of life?
a. TGA
b. TOF
c. Tricuspid atresia
d. Pulmonary atresia

71. The following are characteristic of Malassezia, except:


a. Cluster of thick-walled blastopores with hyphae
b. Spaghetti and meatballs appearance
c. Has predilection for sebum rich areas of the skin
d. Flucytosine is the treatment

72. Which organism most commonly causing fungemia?


a. Malassezia globosa
b. Malassezia furfur
c. Malassezia restricta
d. Malasseza pachydermatitis

73. This represents the tissue phase of Candid:


a. Blastopores
b. Yeast cells
c. Pseudomycelium
d. Pseudohyphae
74. A 6-year-old boy who is a dog lover was brought to your clinic due to 4 days history of fever and headache with the
appearance of maculopapular rash 2 days prior to consult that spreads rapidly to his entire body. He also complained of
calf pain and abdominal pain. Your impression would be:
a. Fungal dermatitis
b. Leptospirosis
c. Ricketssial infection
d. Tetanus

75. What laboratory test would you request of the above case?
a. Culture
b. Microscopic agglutination test
c. Germ tube test
d. No reliable diagnostic test on its acute stage

76. Your drug of choice of the above case would be:


a. Fluconazole
b. Penicillin
c. Doxycycline
d. Metronidazole

77. Infectious form of Chlamydia pneumonia


a. Reticulate body
b. Elementary body
c. Both
d. Neither

78. A sexually transmitted disease:


a. Syphilis
b. Lymphogranuloma venereum
c. Both
d. Neither

79. Latent phase of acquired syphilis:


a. Painless ulcer
b. Painful ulcer
c. Gumma formation
d. Condylomata lata

80. The following is/are virulence factor of staphylococcus aureus, except:


a. Protein A
b. Coagulase
c. Hemolysin
d. Pyrogenic exotoxin

81. A 2-year-old child was brought to you because of 3 days history of voluminous diarrhea with small flecks of mucus
after eating undercooked shellfish. No associated abdominal nor fever. Your initial impression would be:
a. Campylobacter
b. Escherichia coli
c. Vibrio cholera
d. Salmonella infections

82. What laboratory test would you request on the above case?
a. Blood culture
b. Culture of stool or vomitus on thiosulfate citrate bile salts sucrose sugar
c. Bone marrow culture
d. Methylene-blue stained stool smear
83. How will you manage your patient on the above case?
a. Give antibiotics alone
b. Active immunization
c. Give oral rehydration therapy alone
d. A and C

84. A 9-month-old infant was brought in your clinic due to 7 days history of paroxysmal cough associated with gagging
and facial cyanosis. What is the etiologic agent of this case?
a. Hemophilus influenza
b. Bordetella pertussis
c. Streptococcus pneumonia
d. Mycoplasma pneumonia

85. What laboratory test would you request for the above case?
a. Complete blood count
b. Latex particle agglutination test
c. Culture from nose and throat using selective medium
d. No specific laboratory tests

86. The following are the Major criteria of Staphylococcal Toxic Shock Syndrome, except:
a. Acute fever, temp >38.8
b. Hypotension
c. Renal impairment
d. Rash

87. Which of the following meets the criteria in diagnosing Acute Rheumatic Fever?
a. Carditis, Arthralgia and shortened PR interval
b. Chorea, Arthralgia, Elevated ESR and positive rapid streptococcal antigen test
c. Arthralgia, Fever and decrease streptococcal antibody titer
d. Fever, Elevated ESR and CRP

88. A 2-day old infant delivered at home by a traditional birth attendant, was brought to the emergency room due to poor
suck, excessive crying and severe muscular spasm noted 1 day prior to consult. How will you manage the patient?
a. Give antitoxin
b. Antibiotic of choice is Ceftriaxone
c. Antibiotic therapy not part of the treatment
d. A and C

89. Your diagnosis of the above case is:


a. Infant botulism
b. Neonatal tetanus
c. Congenital syphilis
d. Neonatal bacterial sepsis

90. Criteria in Hutchinson’s triad, except:


a. Hutchinson teeth
b. Frontal boss of Parrott
c. Interstitial keratitis
d. 8th nerve deafness

91. Tooth eruption generally begins at what age?


a. 0-1 month
b. 6-8 months
c. 2-4 months
d. 10-14 months
92. The SMR stage of a female adolescent who presents with coarse abundant curly pubic hair and an areola and papilla
with a secondary mound is a:
a. SMR 1
b. SMR 3
c. SMR 2
d. SMR 4

93. A 4-year-old child will have a weight of


a. 16 kg
b. 15 kg
c. 14 kg
d. 18 kg

94. An 8 months old infant is expected to have


a. 3 teeth
b. 1 tooth
c. 2 teeth
d. 6 teeth

95. Breast fed infants should receive iron supplements by


a. 1-2 months of age
b. 4-6 months of age
c. 2-4 months of age
d. 6-9 months of age

96. Handedness is usually determined by what age?


a. 6-12 months
b. 20-24 months
c. 15-18 months
d. 36-48 months

97. A parent brings a toddler, 19 months old to the clinic for a regular check-up. When palpating the fontanels, what
should the doctor expect?
a. Closed anterior and open posterior fontanels
b. Open anterior and closed posterior fontanels
c. Closed anterior and posterior fontanels
d. Open anterior and posterior fontanels

98. Delayed eruption is considered when there is no teeth at approximately:


a. 6 months
b. 10 months
c. 9 months
d. 13 months

99. Which of the following intelligence quotient (IQ) would signify severe mental retardation?
a. 80
b. 45
c. 60
d. 30

100. Gold standard for the diagnosis of TB is


a. Chest X-ray
b. Sputum AFB
c. Sputum Culture
d. Bactec TB 460
Cagayan State University
College of Medicine and Surgery
Written Revalida
Department of Pediatrics

Name ​ ​ ​Date

Score ​ ​

MULTIPLE CHOICE. Write letter of the correct answer before each number. No erasures.

_________ 1. This is the first bone to ossify in a fetus.


​a. Wrist ​ ​c. Sternum
​b. Clavicle ​ ​ ​d. Femur

_________ 2. This is the most common congenital heart disease comprising 45%.
​a. ASD ​ ​c. VSD
​b. PDA ​ ​d. TOF

_________ 3. This appears as “Egg on a string” appearance on Chest X-ray.


​a. TGA ​ ​c. Coarctation of the aorta
​b. TAPVR ​ ​d. TOF

_________ 4. It appear as “String Sign” on abdominal X ray


​a. Pyloric Stenosis ​ ​c. Intussussception
​b. Duodenal atresia ​ ​d. Ileal atresia

_________ 5. This symptom alone is adequate to satisfy the Jones criteria.


​a. Carditis ​c. Erythema multiforme
​b. Sydenham’s chorea ​d. Arthritis

_________ 6. ​What is the most common cause of Furuncles and Carbuncles?


​a. Staphylococcus aureus ​c. Streptococcus pyogenes
​b. Staphylococcus epidermidis d​ . all of the above

_________ 7. Infants of diabetic mothers are watched out for


​a. Sepsis ​c. Bleeding
​b. Respiratory distress syndrome ​d. Hypoglycemia
_________ 8. Staphylococcal Infections are usually transmitted thru
​a. Droplets ​c. Contaminated food and water
​b. Mosquitoes ​d. Persons hands

_________ 9. The earliest indicator of Pneumonia in a child of 6 months is


​a. Wheezing ​c. RR and above 50/min
​b. stridor ​d. chest indrawing
_________ 10. The most common cause of diarrheal disease in the first 2 years of life
​a. Rotavirus ​c. E. coli
​b. Vibrio cholera ​d. Amoebiasis

_________ 11. A 10 year old student develops frequent episodes of spontaneous blank staring for 5
seconds. Aside from that she is normal, she most likely has
​a. Benign Rolandic epilepsy ​c. Atomic Seizures
​b. Complex partial seizure ​d. Simple absence (petit mal)

_________ 12. What is the most common serious complication of Kawasaki disease?
​a. Meningitis ​c. Hepatitis
​b. Renal disease ​d. Coronary aneurysm

_________ 13. A 10 year old girl has lethargy, change of sensorium, headache, nausea, vomiting and
fever. Five years ago, she was hospitalized because of measles pneumonia. Her condition is
probably
​a. Meningitis ​c. Guillain Barre Syndrome
​b. Subacute Sclerosing panencephalitis ​d. Japanese B encephalitis

_________ 14. A 2y/o boy has vomiting and cyanosis following bouts of forceful coughing of 3 weeks
duration. His WBC is 20,000; Neutrophils of 10; lymphocytes 90. The most likely diagnosis is
​a. Tuberculosis ​c. Pertussis
​b. Diphtheria ​d. Pneumonia

_________ 15. A newborn with Congenital Rubella syndrome continuously shed the virus until
​a. for the rest of his life ​c. until 3 years old
​b. until treatment is given ​d. until 1 year old

_________ 16. Hemophilus influenza infections are most common between


​a. 3 mos. up to 5 years ​ ​c. 5 to 12 years old
​b. birth to 2 mos. ​d. 4 years to 10 years old

_________ 17. Which of the following is abnormal in a Newborn


​a. Red tinged diaper ​c. Epstein pearls
​b. Mongolian spot ​d. Grunting

_________ 18. Dental caries in the upper incisors is most commonly suggestive of
​a. Insufficient fluoride ​c. Milk bottle syndrome
​b. Pacifier over use ​d. Infrequent toothbrushing

_________ 19. A 5 y/o boy has multiple papules with excoriations over the finger webs, axilla, wrists,
buttocks and genitals which is especially pruritic at night. His other sibling also complaints of
the same condition. This is likely a case of
​a. Pinworm disease ​c. Pediculosis corporis
​b. Atopic Dermatitis ​d. Scabies

_________ 20. Immunoglobulin that passes the placenta


​a. Ig A ​c. Ig E
​b. Ig G ​ ​d. Ig M

_________ 21. Immunoglobulin that is found in allergic reaction


​a. Ig A ​c. Ig E
​b. Ig G ​d. Ig M

_________ 22. If the mother is HbsAg (+), the baby should be given
a. HB Ig within 12 hours after birth
b. HB vaccine within 7 days
c. A & B
d. None of the above

_________ 23. This is characterized as hypertension of the upper extremities


​a. Coarctation of the Aorta ​c. TAPVR
​b. TOF ​d. TGA

_________ 24. Positive Mantoux test in a previously vaccinated child


​a. 20 mm induration ​c. < 10mm induration
​b. 10 mm induration ​ ​d. > 20mm induration
_________ 25. The normal average weight (in kg) of a Filipino newborn is
a. 2-4 kg. ​ ​c. 5-6 kg.
b. 1-2 kg. ​d. 4-5 kg.

_________ 26. Drug of choice for the treatment of typhoid fever is


​a. Ampicillin ​ ​c. Chloramphenicol
​b. Tetracycline ​d. Amoxicillin

_________ 27. An absolute contraindication to breastfeeding is


​a. Mastitis ​c. Cigarette smoking
​b. Breast tumor ​d. Active TB in the mother

_________ 28. The clinical picture of “Marasmus” originate from the following conditions Except
a. Inadequate caloric intake
b. Proper feeding habits
c. Disturbed parent-child relationship
d. Metabolic abnormalities

st
_________ 29. The physiologic weight loss for the 1 7 days in a newborn is
​a. 20% ​c. 5%
​b. 10% ​ ​d. none

_________ 30. Rheumatic fever is frequently observed in the age group from
​a. 5-15 years ​ ​c. 16-18 years
​b. 1-2 years ​d. 2-5 years

_________ 31. Continues ear discharges longer than 2 weeks indicate


​a. Otitis media ​ ​c. Mastoiditis
​b. Tympanitis ​d. Otitis Externa
_________ 32. A 15y/o girl developed a mild sore throat, low grade fever, diffuse maculopapular rash
and later noted to have tender swelling of the wrist and redness of the eyes with tender
enlarged posterior cervical and occipital lymph nodes. Your most likely diagnosis
​a. Rubella ​c. Roseola
​b. Rubeola ​d. Mumps

_________ 33. This is the most common adverse effect of Isoniazid


​a. jaundice ​c. fever
​b. peripheral neuritis ​ ​d. vomiting

_________ 34. Bronchiolitis is usually caused by


​a. Staphylococcus ​c. RSV
​b. H. influenzae ​d. Echo virus

_________ 35. True of Pneumocystic carinii infection in children


a. Extrapulmonary infection is common
b. Important pathogen in children with AIDS
c. The organism can be cultured from sputum
d. It is common cause of pneumonia in day care centers

_________ 36. Pleurisy is an accumulation of fluid in the pleural spaces that is most often associated
with
​a. H. influenzae ​c. M. tuberculosis
​b. Staphylococcus ​d. Pneumoccocus

_________ 37. Treatment for Idiopathic Thrombocytopenic Purpura


​a. Platelet Transfusion ​c. Exchange transfusion
​b. Prednisone ​ ​d. PRBC transfusion

_________ 38. Bacterial Endocarditis due to Streptococcus viridans is best treated with
​a. Penicillin G ​c. Ampicillin
​b. Vancomycin ​d. Gentamicin

_________ 39. Cyanosis in Newborns may be caused by all of the following Except
​a. Tetralogy of Fallot ​c. Tricuspid atresia
​b. Patent Ductus arteriosus ​d. Persistent Pulmonary Hypertension

_________ 40. In females, 75-90% of Urinary Tract Infection are caused by


​a. Proteus sp. ​c. E. coli
​b. S. saprophyticus ​d. Klebsiella

_________ 41. Duration of secondary prophylaxis for Rheumatic fever is


​a. 6 mos. ​c. 1 year
​b. 5 years ​ ​d. 2 years
_________ 42. Drug/s of choice in the treatment of patients with recurrent complex febrile seizures
​a. Phenobarbital ​c. Valproic Acid
​b. Diazepam ​d. all of the above

_________ 43. Pneumothorax can best be diagnosed by a chest x-ray. The following findings is
typical in this condition
​a. Cysticl luscensis ​c. Air-fluid level
​b. Wedge-shaped gravity ​d. Absence of lung markings

_________ 44. The most common cause of Gross hematuria in pediatrics


​a. AGN ​c. Trauma to the Urinary Tract
​b. UTI ​d. Ig A Nephropathy

_________ 45. A 10 year old child developed an acute attack of asthma. The drug that would afford
immediate relief for this would be
​a. Nebulized corticosteroid ​c. inhaled corticosteroid
​b. Nebulized B2 agonist ​ ​d. inhaled cromolyn Na

_________ 46. The bone most frequently fractured during labor and delivery in the newborn
​a. Femur ​c. Fibula
​b. Tibia ​d. Clavicle

_________ 47. Which of the following may not necessarily be present at birth
​a. grasp ​c. Plantar reflex
​b. Moro reflex ​ ​d. Babinski reflex

_________ 48. A micronutrient that helps in the formation of red blood cells, prevent serious neural
tube defects of the spinal cord and brain to
​a. Iodine ​c. Zinc
​b. Iron ​d. Folate

_________49. Management of hyperbilirubinemia of the Newborn is by


​a. phototherapy ​c. Phenobarbital
​b. Exchange transfusion ​d. all of the above

_________50. A 7 month-old male infant was brought in because of bloody stools. History revealed
that 5 days age, he had LBM. PPE: (+) elongated mass our (L) lower quadrant of the abdomen.
The most likely diagnosis is
​a. Polyp ​c. Salmonellosis
​b. Amoebiasis ​d. Intusussception

_________ 51. Ecthyma gangrenosum is a cutaneous lesion associated with


a. Group A B hemolytic Streptococcus
b. Pseudomonas aeruginous
c. Staphylococcus aureus
d. Neisseria meningitidis

_________52. Drug of choice for Pseudomonas aeruginosa


​a. Erythromycin ​c. Ceftazidime
​b. Streptomycin ​d. Ceftriaxone

_________53. Which of the following drugs crosses the blood brain barrier even in the absence of
inflammation.
​a. Amikacin ​c. Cephalexin
​b. Chloramphenicol ​ ​d. Cefuroxime
_________54. The most common malignant neoplasm in children is
​a. Hodgkin’s disease ​c. Acute Lymphoblastic Leukemia
​b. Neuroblastoma ​d. Wilms’ tumor

_________55. Acute epiglotittis is a life threatening infection in children. This is generally associated
with
​a. Hemophilus influenza ​c. Streptococcal pneumonia
​b. Streptococcus pyogenes ​d. Staph. aureus

_________56. Hemorrhagic disease of the newborn is a deficiency seen more in breastfed infant
because of
a. lower vitamin C in human milk ​c. lower minerals in human milk
b. lower Vitamin K in human milk ​ ​d. none of the above

_________57. The rash of unmodified measles appear first


a. at the chest and back
b. at the hairline, forehead and behind the ear lobe
c. in the interdigital area
d. on the dorsum of the hands and feet

_________58. A subdural effusion most commonly accompanies meningitis cause by


​a. Streptococcus pneumonia ​c. E. coli
​b. Hemophilus influenzae ​ ​d. Neisseria meningilidis

_________59. Of the anthropometric indicators of nutritional status, measurement of the following is


a better criterion because it is independent on excess fluid or fat
​a. Head circumference ​c. Triceps skin fold thickness
​b. Height ​ ​d. Weight

_________60. Congestive heart failure is ready observed in


​a. Tetralogy of Fallot ​c. TGA
​b. VSD ​d. PDA

_________61. Drug of choice for Leptospirosis is


​a. Penicillin ​ ​c. Sulfisoxazole
​b. Chloramphenicol ​d. Tetracycline

_________62. The passage of feces into inappropriate places after bowel control has been established
is known as
​a. Constipation ​c. Enuresis
​b. Encopresis ​ ​d. Obstipation

_________63. A child who discharges urine involuntarily after the age at which bladder control should
have been established is known as
​a. Polyuria ​c. Enuresis
​b. Dysuria ​d. Polydypsia

_________64. A newborn infant who fails to pass meconium within 48 hrs. and later presents with
abdominal distention and dilatation of proximal colon is probably
​a. Hirschsprung’s disease ​ ​c. Meconium ileus
​b. Jejunal atresia ​d. Malrotation

_________65. The prevention of extrapulmonary tuberculosis is administered by the use of the BCG
vaccine. This vaccine is best administered by
​a. Subcutaneous ​c. Intramuscular
​b. Intradermal ​ ​d. Oral

_________66. The best measure for prevention of infection among neonates in a nursery is
​a. Use of Alcohol ​c. Discard used blankets
​b. Use of Betadine ​d. Enforce correct hand to elbow washing

_________67. This viral infection may cause congenital heart disease., microcephaly, cataracts and
deafness, if acquired in the first trimester of pregnancy.
​a. Epstein Barr ​c. Varicella
​b. Rubella ​ ​d. Parvovirus B19

_________68. The following findings is Abnormal in a normal newborn


a. liver edge 1-2cm below costal margin ​
b. Barrel-shaped chest
c. Umbilical cord with one artery 2 veins
d. Spleen tip barely palpable

_________69. At what age can a child identify his gender ?


a. When the child is already attracted to the opposite sex
b. At 1 ½ y/o
c. At 4 y/o
d. At 3 y/o

_________70. The first permanent tooth to erupt is


​a. central incisors ​ ​c. molar
​b. cuspids ​d. premolars

_________71. An HIV patient can be given


​a. DPT ​ ​c. both
​b. MMR ​d. none

_________72. Normal newborn Except:


​a. HC 28 cm ​ ​c. wt. of 3 kg
​b. CC 38cm ​d. birth length of 48cm

_________73. Draws a man with 2 parts, goes to toilet alone, tells a story, role playing
​a. 2y/o ​c. 4 y/o
​b. 3 y/o ​d. 5 y/o

_________74. First sign of Puberty in males


​a. Testicular enlargement ​ ​c. Axillary hair
​b. Penile enlargement ​d. facial hair
_________75. Vitamin deficiency found in the developing countries especially in the Philippines.
​a. Thiamine deficiency ​c. Vitamin C deficiency ​
​b. Vitamin A deficiency ​d. Vitamin B12

_________76. First sign of Puberty in Females


​a. Thelarche ​ ​c. appearance of pubic hair
​b. Menarche ​d. growth spurt

_________77. Human colostrum has


​a. more carbohydrates ​c. less mineral
​b. more fats ​d. more protein

_________78. The murmur of a 2 year old is continuous or machinery type heard in left upper sternal
border. The diagnosis most likely is:
​a. Tetralogy of Fallot ​c. Patent Ductus Arterious ​
b. Atrial Septal Defect ​d. Ventricular Septal defect

_________79. A 2 year old child developed fever, headache, vomiting and nuchal rigidity following a
bout of lower respiratory infection of the possible causes of bacterial meningitis, the following
is more likely:
​a. N. meningitidis ​c. Salmonella
​b. Streptococcus pneumoniae ​d. Staphylococcus aureus

_________80. A 2 months old infant comes in with jaundice since 1 week old, highly colored urine,
acholic stools. Liver edge is palpated below Rt. Subcostal margin. The most likely diagnosis is
​a. Biliary Atresia ​c. Viral Hepatitis
​b. Hepatoblastoma ​d. Inspissated bile syndrome

_________81. A cephalhematoma is best differentiated from caput succedaneum by:


a. a normal neurologic examination
b. absence of history of prolonged labor
c. swelling does not cross suture line
d. all of the above

_________82. Status epilepticus is continuous clinical seizure which lasts more than
​a. 3 hrs ​c. 30 minutes
​b. 15 minutes ​ ​d. 60 minutes

_________83. Nutritional deprivation particularly during the first 3 years of life may have serious and
irreversible effect on growth and development of:
​a. Bone ​c. kidney and lung
​b. Brain ​d. muscular and subcutaneous

_________84. A deficiency of fatty acid causes failure of growth and the production of dry, thick scaly
skin with desquamation and intertrigo:
​a. Linoleic acid ​c. Palmitin
​b. caproic acid ​d. Butyric acid

_________85. The single most important cause of preventable brain damage and mental retardation is
the deficiency of the following nutrient:
​a. Zinc ​c. Iodine
​b. Iron ​d. Vitamin A

_________86. The drug of choice for the treatment of cholera


​a. Ampicillin ​c. Tetracycline
​b. Oxacillin ​d. Amoxicillin

_________87. A child who can skip with alternating feet, stand on one foot for 10 seconds, and tie his
own shoelaces has reached the developmental motor level of
​a. 2 years ​c. 6 years
​b. 4 years ​d. 8 years

_________88. This is the most common congenital viral infection


​a. Varicella ​ ​c. CMV
​b. Rubella ​d. Parvovirus

_________89. Fever is remittent if:


a. daily temperature elevation return to normal
b. daily temperature return to baseline but above the normal temperature
0
c. The fluctuation of temperature elevation is <3 C
d. None of the above

_________90. Diagnosis of URTI can be made if the urine culture yielded gram (-) enteric rods of:
a. 100 col/ml of urine by suprapubic aspiration
b. 10,000 col/ml of urine obtained by catheterization
c. 100,000 col/ml of clean catched urine ​
d. All of the above

_________91. The most common cause of Viral Pneumonia in infancy is


​a. Parainfluenza ​c. RSV
​b. Influenza ​d. Adenovirus

_________92. The most common bacterial pathogen for osteomyelitis is


​a. H. influenzae ​c. S. pneumonia
​b. S. aureus ​ ​d. S. pyogenes

_________93. The combination of hematuria and proteinuria usually indicates
​a. nephrolithiasis ​c. glomerulonephritis
​b. renal tumor ​d. urinary tract infection

_________94. At one year old, how many times should a baby’s weight have increased?
a. 4 ​c. 2
b. 3 ​ ​d. 5

_________95. A baby with diarrhea should not be given


​a. artificial milk ​c. fresh fruit juices
​b. rice gruel ​d. breastmilk

_________96. The head circumference and chest circumference approximate at


​a. 4 months ​ ​c. 24 months
​b. 18 months ​d. 12 months
_________97. Majority of term infants will have passed their first meconium:
​a. within a few minutes of life ​c. by 24 hrs. of life ​
​b. during the process of birth ​d. by 6-12 hrs. of life

_________98. Hemophilia A is the most frequent type of hemophilia disorders and is characterized
by deficiency of
​a. Factor VIII ​ ​c. Factor VII
​b. Factor VI ​d. Factor X

_________99. An 8y/o comes to the ER and diagnosed as Poststreptococcal Glomerulonephritis with


a blood pressure of 160/110. Your first drug of choice to give is
​a. Nifedipine sublingual ​c. hydralazine
​b. Furosemide ​ ​d. Spironolactone
_________100. An extrapulmonary tuberculosis infection in children affecting the superficial lymph
node is
​a. Cervical lymphadenitis ​c. Regional Lymphadenitis
​b. BCG induced adenitis ​d. Scrofula

​END
​ 1. The specific treatment of Respiratory Distress Syndrome is
a​ . Oxygen c​ . Antibiotics
​ . Surfactant
b ​ . Continuous Positive Airway Pressure
d

​ 2. The following factors predispose a neonate to develop hyaline membrane disease


Except
​a. Maternal Diabetes ​c. Asphyxia Neonatorum
​b. Post Maturity ​d. Caesarian section delivery

​ 3. The most commonly fractured bone in the newborn in difficult deliveries is the
a​ . Cranial bones c​ . Tibia
​ . Femur
b d​ . Clavicle

​ 4. A scaphoid abdomen in the Newborn suggests


a​ . Emphysema c​ . Diaphragmatic hernia
​ . Hyaline membrane disease d​ . Malrotation of the intestine
b
​ 5. The most serious complication of hyperbilirubinemia in Newborn is
a​ . Heart failure c​ . Kernicterus
​ . Severe anemia
b d​ . Sepsis

​ 6. The usual source of Tetanus Neonatorum is


a​ . Circumcision wound c​ . Vaccination
​ . Umbilical stump
b d​ . All of the above

ENDOCRINE & METABOLIC CONDITION

​ 1. Hypopituitarism Except
​a. Cherubic face ​c. Small penis
​b. Short neck and webbing ​d. Delayed dental development

​ 2. Characteristics Diabetes Insipidus Except:


​a. Urine specific gravity 1.001-1.005 ​c. Dehydration is not frequent
​b. Maybe due to encephalitis ​d. None of the above

​ 3. True of Anterior Pituitary Gland Except:


​a. Secretes ACTH responsible for hyperpigmentation
​b. Responsible for the stimulation of the nipple to breastfeed
​c. Secretes oxytocin responsible for milk-letdown reflex
​d. Regulation of hormone secretion is caused by negative feedback mechanism

​ 4. Characterized SIADH
​a. Polydipsia ​c. Polyuria
​b. Water intoxication ​ ​d. Extreme thirst

​ 5. Congenital Hypothyroidism Except


​a. Prolonged Jaundice ​c. Due to post hypothyroidectomy
​b. Due to thyroid dyshormogenesis ​d. Constipation

​ 6. Signs and symptoms of Congenital Hypothyroidism


a​ . Constipation c​ . Poor feeding
​ . Hypernatremia
b ​ . A&C
d

​ 7. Drug of choice for Congenital Hypothyroidism


a​ . PTU c​ . Na Levothyroxine
​ . Methimazole
b d​ . Tapazole

CARDIOLOGY
______1. Cyanosis with clubbing of fingers and toes is seen in

a. Left to Right shunt ​c. Congestive heart failure


b. Right to Left shunt ​d. None of the above

______2. A history of stroke is more commonly seen in


a. Cyanotic heart disease ​c. Obstructive lesion
b. Acyanotic heart disease ​d. None of the above

______3. The most common cause Hypertension in children and adolescent


a. Cardiovascular in nature ​c. CNS in nature
b. Renovascular in nature ​d. Endocrine in nature

______4. Pharmacologic closure of the ductus has been reported in Premature if treated before 10
days
​ of age with
a. Furosemide ​c. Prostaglandin
b. Indomethacin ​ ​d. Aspirin

______5. Chest x ray configuration of “egg on its side”


a. TGA ​ ​c. TOF
b. TAPVR ​d. Coarctation of the Aorta

______6. As in most patients with Congenital heart disease,squatting is a frequent manifestation


a. PDA ​ ​c. TOF
b. VSD ​d. Coarctation of the aorta

NEUROLOGY

______1. The absolute contraindication of a lumbar puncture is


a. Suspicion of CNS disease ​c. Increase intracranial pressure
b. Bleeding diathesis ​d. Presence of infection over the site of
puncture

______2. Malignant tertian Malaria that causes CNS infection is


a. Plasmodium falciparum ​ ​c. Plasmodium vivax
b. Plasmodium ovale ​d. Plasmodium malariae

______3. The following are all manifestations of poliomyelitis, Except


a. Ascending symmetrical paralysis c. Absent deep tendon reflexes
b. Motor paralysis ​ d. Positive tripod sign

______4. Sudden, brief, shock-like contraction of muscles,or jacknife seizure


a. Petit mal ​c. Atonic seizure
b. Myoclonic seizure ​d. Tonic-clonic seizure

______5. Jake suddenly woke up crying at night pointing incoherently at the night lamp. His mother
lulled him back to sleep. He has no recall of the incident the next morning. What did he have?
a. Somnambulism ​c. Night terror
b. Nightmare ​d. Bed wetting

______6. A continous seizure more than 30 minutes without regaining consciousness


a. Status epilepticus ​ ​c. Infantile seizure
b. Grand mal ​d. None of the above

GASTROINTESTINAL TRACT

______1. A 2 week old male infant was seen at the ER with projectile vomiting for 5 days, non
billous. You noted visible peristalsis, with palpable tumor in the midline. Your main
consideration is
a. Gastroesophageal Reflux
b. Congenital Hypertrophic Pyloric stenosis
c. Tracheoesophageal Fistula
d. None of the above

______2. Causes of upper GI bleeding in infancy Except


a. Ingestion of red food dyes ​c. Infectious diarrhea
b. Gastritis ​d. Esophagitis

______3. Failure of passage of meconium during the 1st 24 hours of life should alert the physician to
the possibility of ____ Except
a. Hirschsprung’s disease ​c. Anorectal malformation
b. Atresia ​d. None of the above

______4. The best time for operation for cleft lip


a. 6 months-2 years ​ ​c. 2-4 years old
b. 1-2 months ​d. At birth

______5. The most common cause of halithosis in children


a. Dental caries ​ ​c. Bronchiectasis
b. Lung abscess ​d. TB

______6.Intermittent gastrointestinal bleeding without abdominal pain is the most common symptom
of
a. Meckel’s diverticulum ​c. Meconium plug syndrome
b. Hypoplastic left colon ​d. Hirschsprung disease

DEVELOPMENTAL AND GENETIC DISORDERS

_____1. Best known example of X-linked recessive inheritance


a. Fragile X ​c. Marfan’s syndrome
b. Albinism ​d. Hemophilia A and B

_____2. What is the most common cause of abnormalities in chromosome number?


a. Monosomies ​c. Aneuploidy
b. Trisomies ​ ​d. Translocation

_____3. This is the most common cause of mental retardation


a. Down syndrome ​c. Edward syndrome
b. Trisomy 13 ​d. Cri-du-chat syndrome
c.

_____4. Classical features of short stature, webbed neck, cubitus valgus,and lack of secondary
Sex characteristics
a. Klinefelter syndrome ​c. Turner syndrome
b. Triple female ​d. Fragile X syndrome

_____5. These are examples of Multifactorial inheritance Except


a. Congenital heart disease ​c. Cleft palate
b. Clubfoot ​d. None of the above

_____6. These are Autosomal dominant trait Except


a. Night blindness ​c. Turner’s syndrome
b. Huntington’s chorea ​d. Marfan’s syndrome

_____7. This is the 2nd most common cause of mental retardation


a. Down syndrome ​c. Patau syndrome
b. Fragile X syndrome ​d. Cri-du-chat syndrome

_____8. XO chromosome is found in


a. Triple female ​c. Fragile X
b. Klinefelter syndrome ​d. Turner syndrome

RESPIRATORY

_____1. A 9 year old boy swallowed a coin. On antero-posterior chest film the coin is seen facing
you. Where is the coin lodge?
a. Esophagus ​ ​c. Trachea
b. Larynx ​d. Right bronchus
_____2. This condition is confirmed when a no. 8 feeding catheter cannot pass through either nostrils
into the oropharynx
a. Tracheoesophageal fistula ​c. Pierre Robin syndrome
b. Diaphragmatic hernia ​d. Choanal atresia
_____3. Epiglottitis is a rapidly progressive disease that leads to respiratory failure and is caused by
a. Streptococcal bacteria ​c. E. coli
b. Staphylococcus aureus ​d. Haemophilus influenzae

_____4. Early cyanosis and respiratory distress, scaphoid abdomen, barrel-shaped chest and
dextrocardia suggest
a. Bronchiolitis ​
​b. Congenital diaphragmatic hernia
c. Sequestration of the lung
d. Congenital cystic adenomatous malformation

_____5. Atypical pneumonia is commonly caused by Except


a. Mycoplasma pneumoniae ​c. Legionella pneumophila
b. Chlamydia pneumoniae ​d. Grp B streptococcus
_____6. The following are the most common bacterial pathogens causing pneumonia in Newborn
Except
a. H. Influenzae ​c. Gram negative bacilli
b. Grp B streptococcus ​d. Chlamydia

ALLERGIC/ IMMUNOLOGIC CONDITIONS

______1. Triad of Bronchial asthma Except


a. Bronchodilation ​ ​c. Mucoid edema
b. Smooth muscle spasms ​d. Plugging with tenacious mucus
______2. Dark circles under the eyes due to venous stasis resulting from blood flow obstruction
caused by edema of nasal mucous membranes
a. Rabbit Nose ​c. Allergic salute
b. Allergic shiners ​ ​d. None of the above

​ 3. Drug of choice for early treatment of Atopic child


​a. Ranitidine ​ c​ . Cetirizine
​b. Hydroxyzine d​ . Mequitazine

​ 4. The following are events in Acute Asthma attack Except


​a. Decrease FEV1 c​ . Hyper inflation of the lungs
​b. Prolonged inspiratory time d​ . Increase work of breathing

​ 5. Severe Acute Asthma that is unresponsive to the usual appropriate therapy


​a. Acute Asthma Exacerbation c​ . Status Epilepticus
​b. Status Asthmaticus d​ . Pulsus paradoxus

​ 6. Atopic Dermatitis characterized by erythematous, prurituc, weeping dermatitis in the cheek,


w/ heavy scaling of the scalp
a. Childhood stage w/ AD ​c. Adult stage w/ AD
b. Infantile stage w/ AD ​d. All of the above

​ 7. Food allergy is the most common clinical manifestation of


​a. vomiting c​ . GI Bleeding
​b. diarrhea ​d. abdominal pain

NUTRITIONAL PROBLEMS

​ 1. True of Marasmus Except


​a. Diet due to both low in protein & calories ​c. Pot belly
​b. Apathetic ​d. Edematous

​ 2. Signs of Kwashiorkor Except


​a. “ flag sign “ ​c. Moon Face
​b. “Winged scapulae” ​ ​d. Flaky paint rash

​ 3. Bitot’ spots is eye sign of


​a. Vitamin D Deficiency ​c. Vitamin A Deficiency
​b. Vitamin B Deficiency ​c. Vitamin E Deficiency

​ 4. Cheilosis, glossitis, atrophic lingual papillae are signs of


a​ . Niacin deficiency c​ . Vitamin A deficiency
b​ . Thiamine deficiency ​ . Riboflavin deficiency
d

​ 5. Irritability, depression, somnolence and seizures are signs of


a​ . Pyridoxine deficiency c​ . Riboflavin deficiency
​ . Thiamine deficiency ​
b d​ . Niacin deficiency

BASIC CHILD CARE

​ 1. Visual acuity of 20/20 is reached at age


a​ . 6 c​ . 8
​ .7
b d​ . 5

​ 2. The 1 sign of puberty in male is


st

​a. Change in voice ​c. Appearance of axillary hair


b​ . Appearance of public hair ​d. Enlargement of testes

​ 3. Which of the following is synthesized in the gastrointestinal tract?


a​ . Vitamin A c​ . Vitamin D
b​ . Vitamin C ​ . Vitamin K
d

​ 4. A child with Pica most likely has which deficiency?


​a. Iodine ​c. Magnesium
​b. Calcium ​d. Iron

​ 5. A hepatitis B positive mother just gave birth. Which of the following will you administer to
the baby?
a. Hepatitis B vaccine ​c. Both A & B
b. Hepatitis B immunoglobulin ​d. Tetanus vaccine
​ 6. Immunoglobulin that crosses the placenta
a​ . Ig A c​ . Ig D
​ . Ig G
b ​ d​ . Ig E

​ 7. What is the sodium content of the WHO oral rehydration solution?


a​ . 25 mmol/L c​ . 50 mmol/L
​ . 20 mmol/L
b ​ . 90 mmol/L
d

​ 8. What is the age in month of a child who can roll over, reach for objects, sit on tripod
​ position, Bubbles when caretaker talks to her and has a good head control?
​a. 3 ​c. 5
​b. 4 ​d. 6

​ 9. During a Newborn’s 1 week of life, how much weight loss is expected due to excretion of
st

excess extravascular fluid and poor intake?


​a. 5% ​c. 15%
​b. 10% ​ ​d. 20%

​ 10. It is considered as the most important cause of brain damage and mental retardation
a​ . Iron deficiency c​ . Vitamin A deficiency
​ . Iodine deficiency
b d​ . Zinc deficiency

​ 11. It is considered as the most common nutritional disorder in the world.


a​ . Iodine deficiency c​ . Vitamin A deficiency
​ . Iron deficiency
b d​ . Zinc deficiency

​ 12. A Filipino Newborn usually measures the following at birth


a​ . wt 3.5kg L ​ t . 50cm H​ C 20cm
​ ​ . wt 3kg L​ t . 50cm H
b ​ C 35cm
​ c​ . wt 2.5kg L​ t . 45cm H​ C 33cm
​ . wt 3kg L​ t . 45cm H​ C 12.14cm
d

INFECTIOUS DISEASE
​ 1. Pathognomonic exanthema of measles
a​ . Conjunctivitis c​ . Koplik spots
​ . Coryza
b d​ . Cough

​ 2. Purpuric skin lesions in Congenital Rubella is described as


a​ . “glove and stocking” c​ . “slapped cheek”
​ . “Blueberry muffin” ​
b d​ . None of the above

​ 3. The occurrence of Congenital defects is as high as 85% if infection occurs during _____ of
gestation
​a. 2nd
month ​ ​c. 1 st
month
​b. 3rd
month ​d. 4
th
month

​ 4. The most common cause of diarrhea in infants and children between 4 and 24 month of age
a​ . Salmonella c​ . Cholera
​ . Rotavirus ​
b d​ . Adenovirus

​ 5. The most frequent cause of the common colds


a​ . Influenza B c​ . Rhinovirus
​ . Para influenza d​ . Adenovirus
b

​ 6. The most common cause of Bronchiolitis in infants


​ . Respiratory Syncitial Virus
a c​ . Streptococcus pneumonia
​ . Hemophilus influenzae
b d​ . Rhinoviruses

​ 7. Cough in Pertussis lasts for 6 to 8 weeks, and the drug of choice is


a​ . Pen G c​ . Cloxacillin
​ . Pen VK
b ​ . Erythromycin
d

HEMATOLOGIC-ONCOLOGIC

​ 1. These are group of hereditary anemias with mutations that affect the synthesis of
​hemoglobin
​a. Hereditary Sperocytosis ​c. Iron deficiency anemia
​b. Thalassemia ​d. none of the above

​ 2. Megaloblastic Anemia is treated with


a​ . Iron c​ . Desferroxamine
​ . Folic acid ​
b d​ . Corticorteroids

​ 3. Aplastic Anemia refers to the failure of the marrow to produce all the marrow cells Except:
​a. Blast cells ​ ​c. Myeloid cells
​b. Red cells ​d. Platelets

​ 4. Physiologic Anemia in Term infants around this age


​ . 1 month old ​
a c​ . 2 month old

​b. 6 months old ​d. 1 year old

​ 5. A 2 month old infant was brought to the ER due to seizure. History revealed that he was
delivered at home uncomplicated by a hilot. No Hx of fall nor fever was noted. PE revealed
pallor with bulging tense anterior fontanel and ptosis on the left. Your main considerations
would be
​a. Acute head injury ​c. Seizure disorder
b. Bacterial Meningitis ​d. APCD

​ 6. These are the Vitamin K dependent clotting factors Except:


​ .V ​
a c​ . VII
​ . II
b d​ . IX

MUSCULOSKELETAL ​

​ 1. A positive Ortolani test means that the hip is


a​ . unstable c​ . normal
​ . dislocated ​
b d​ . none of the above

​ 2. A “sunburst appearance” in the x-ray is characteristic of


a​ . Aneurysmal bone cyst c​ . Ewing’s sarcoma
​ . Osteogenic sarcoma d​ . none of the above
b

​ 3. This syndrome is characterized by joint hypermobility, skin extensibility, early bruising


and soft tissue calcification
​a. Osteogenesis imperfecta ​c. Ehler-Danlos syndrome
​b. Rickets ​d. None of the above

DERMATOLOGIC CONDITION

​ 1. These are small, pinpoint to pinhead, milky, smooth papules on the forehead & cheeks of
newborn
​a. Erythema toxicum ​c. Milia
​b. Acne Neonatorum ​d. none of the above

​ 2. There are linear lesions called “burrows” where the mites are formed, a characterisitic of
a​ . Pediculosis c​ . Cimicosis
​ . Culicosis
b ​ . Scabies
d
​ 3. This result from involvement of skin from underlying tuberculosis of lymph nodes, bones &
joints and Epididymis
a. Miliary TB of the Skin
b. Lupus vulgaris
c. Scrofuloderma
d. None of the above

​ 4. This a hereditary or acquired progressive loss of melanin pigment due to failure of the
​Tyrosinase system of melanocyte in the dermo-epidermal junction.
​a.Albinism ​c. Nevus depigmentosus
​ . Vitiligo
b ​d. Hypomelanosis of Ito

RENAL DISORDER

​ 1. Causative agent for poststreptococcus AGN ​


a​ . Grp. B Streptococcus c​ . Grp. A beta-hemolytic streptococcus
​ . Streptococcus pneumoniae
b d​ . Staphyloccus aureus

​ 2. This complement is decreased in PSAGN


a​ . C4 c​ . C5
​ . CRP
b ​ . C3
d

​ 3. The most common cause of hematuria in children


​ . IgA Nephropathy ​
a c​ . HSP nephropathy
​ . PSAGN
b d​ . None of the above

​ 4. The Causative agent for HNS is


​ . E.coli 0157:H7 ​
a ​c. Salmonella
​b. Shigellosis ​d. None of the above

​ 5. The most common malignant renal neoplasm in children and adolescents


​ . Wilms’ tumor
a ​ c​ . Pheochromocytoma
​ . Neuroblastoma
b d​ . None of the above

​ 6. Which is the 1 line of treatment for Nephrotic Syndrome?


st

​a. Prednisone ​c. Cyclophosphamide


​ . Penicillin ​
b ​d. IV Immunoglobulin

PEDIATRIC EMERGENCIES

​ 1. Accidental ingestion of corrosive agents showed be managed initially of


a​ . Gastric lavage c​ . Administer large amount of water or milk
​b. Neutralize poison ​d. Induce emesis

​ 2. A 3 y/o boy was bitten by a pet dog on the face. The dog has no rabies immunization, which
of the following should be done?
a. observe the dog for 10-14 days
b. give a rabies vaccine to the dog
c. give hyperimmune serum immediately, no vaccine as long as
animal remains normal

d. give hyperimmune serum immediately, followed by vaccine

​ 3. A 2 y/o girl was rushed to the ER because of seizure described as Generalized Tonic
Clonic, w/ loss of consciousness lasting for 10 minutes. You immediately administer
​a. Phenobarbital ​c. Diazepam
​b. Phenytoin ​d. Oxygen

​ 4. A 6 months old by infant was brought to the ER because of vomiting and LBM for 2 days
now. Upon PE, he was drowsy, with sunken eyes, depressed anterior fontanelle and cold
clammy extremities. You immediately administer IVF of
​a. PLRS 30cc/kg for 1 hr ​c. D50.3NaCl 20cc/kg for 1 ½ hr
​b. PLRS 10cc/kg for 1 hr ​d. D50.3 NaCl 10cc/kg for 1 hr

​ 5. A 5 y/o boy was seen at the ER in moderate respiratory distress. History revealed that he
had colds and fever 1 day prior. Then few hour prior to consult, he was noted to be
coughing, PE revealed wheezes all over lung fields. You will do nebulization w/
​a. Corticosteroids ​c. Beta 2 agonist
​b. Cromolyn sodium ​d. All of the above
D C

B
A/ E

E/ B B
GIT (NEW)
1. True of esophageal atresia except
a. 87% occurs with distal TEF
b . History of recurrent pulmonary infection with H-type
c. Aspiration pneumonia is common to all
d. Management is medical
e. A& c
2. Management of GER
a. Lower esophageal sphincter (LES) competent, high LES pressure
b. Competent LES facilities reflux of acid contents into the esophagus
c. Mainly self-limited
d. Poor intake from the start
e. Surgery as initial mode of management
3. Management of GER
a. Thickened oral feeds/orthostatic posturing
b. Funduplication
c. Antacids/prokinetic
d. A&c only
e. AOTA
4. True of achalasia
a . Peristalsis normal in the body of esophagus
b. Abundant argyrophil ganglion cells
c. Failure of LES to relax in response to swallowing
d. A&c only
e. AOTA
5. The ff statements is/are true except
a. In both acid and alkali ingestion gastric lavage and emetics are advisable
b. Acid ingestion - milk or alkali
c. Alkali ingestion - dilute vinegar or citrus juices
d. Antibiotic/ steroids necessary
e. A, b, & c are true
6. Related to management of esophageal varices except
a. May be spontaneous relieved
b . Sclerosing agent
c. Balloon tamponade
d. Surgical shunting as initial management
e. AOTA
7 . Secondary peptic ulcers are
a. Frequently gastric
b. Commonly solitary
c. Equal frequency in males and females
d . A, b, & c are true
e. A & care true
8. True in the pathogenesis of ulcers except
a. Enhanced mucosal prostaglandin level
b . Campylobacter pylori
c. NSAIDs/onti-inflammatory drugs
d. Enhanced acid and pepsin secretion
e. A & c
9. True of peptic ulcer disease
a. Hematemesis - common in primary PUD
b. Scanty GI bleeding - common presentation in children with secondary PUD
c. Melena more in secondary ulceration
d. A & C

e. AOTA
1 0. Management of primary ulcers include
a. Anti cholinergics
b. H2 receptor blockade
c. Re endoscopy after 6 wks treatment
d. A & c only
e. AOTA
11 . Surgical intervention in PUD is warranted in
a. Perforation
b. Intermittent hemorrhage
c. Obstruction
d. A & C

e. AOTA
1 2. True of congenital hypertrophic pyloric stenosis except
a. Increased size of pyloric circular muscle
b. Bigger size of ganglion cells
c. Immaturity of mylateric plexus
d. A&c
e. A&b
1 3. Pyloric stenosis
a. Usually occurs 2 -3 wks old premature female infant
b. lnidicidual has voracious appetite, yet facts to thrive
c. (+) pyloric murmur
d. B&c are correct
e. AOTA
14. Cardinal signs of intestinal obstruction
a. Vomiting
b. Abdominal distention
c. Constipation/obstipation
d. A&c only
e. AOTA
15. Associated radiological findings include
a. Double bubble sign
b. Step ladder appearance
c. Pneumoperitoneum
d. A&b only
e. AOTA
16. Manifestations of intussusception are the ff except
a. Mild squasmodic pain
b. Current jelly stools
c. Strange shaped moss
d. A&c
e. B&c
17. Theories/ etiology of NEC (necrotizing enterocolitis)
a. Gastrointestinal ischemio
b. Underdevelopment of GI immune protection
c. Infectious agents
d . A&c
e. AOTA
1 8. In NEC, the most important indication for surgical intervention
a . Severe blleding
b. Intestinal perforation
c. Abdominal distention
d. A&c
e. B&c
19. Hirschsprung's disease - highly suspected in
a . Failure to poss meconium during 1st 40 of life
b . Progressive abdominal distention
c. Infant refuses feeding
d. Bilious vomitus
e. AOTA
20. Surgical itervetion of Hirschsprungs's disease consists of
a. Segmental biopsy of the involved colon
b. Colostomy
c. Pull through
d . A&c only
e. AOTA
21. True of hepatitis D virus (HDV)
a. Unable to replicate on its own
b. Less pathologic
c. Capable of infection only when activated by HBV
d. A&c only
e. AOTA
22. True of the icteric phase of infectious hepatitis except
a . Prodomol symptoms and fever subsides
b . Liver decreased in size and tenderness
c. lcterus peaks between 3 rd 4 th wk then subsides
d. Identical clinical features in this phase for all infectious forms
e. B&c
23. Hepatitis B immune globulins (HBIG) for peristol exposure
a. HBIG 0.5 ml w /in 1 2 hours of birth
b. HB vaccine 10 mg IM w / in 7 days of life, repeat at 1 & 6 mos
c. HB vaccine 0.5 ml IM w / in 7 days of life, repeat at 1 & 6 mos
d. A, b, or c
e. A & b only
24. Which statement ore true
a . Anti-HBsAg indicates late incubation period or very early stage of acute infection
b. (+) HBsAg indicates late incubation period or very early stage of acne infection
c. (+) HBsAg & (+) HBcAg - highly infectious, signifies early stage of acute infection
d. Positive anti-HBc & anti-HBe represent resolving infection
e. AOTA
25. In chronic liver disease
a. Large inoculums of infectious agents results in chronic infection
b. Individuals with chronic HBV infection have qualitative & quantitative defect in producing
antibody to the virus
c. Altered immunologic mechanism does not allow progression from acute to chronic HBV infection
d. A&c are false
e. A, b & c are true
GIT (OLD)
1. True of esophageal otresio except
a. 37% occurs with distal TEF
b. History of recurrent pulmonary infection with H-type
c. Aspiration pneumonia is common to all
d. Management is medical
e. A & c
2. Management of GER
a. Thickened oral feeds/orthostotic posturing
b. Funduplication
c. Antacids/ prokinetic
d. A & c only
e. All of the above
3. The ff statement is/ ore true except
a. In both acid and olkolin ingestion gastric lavage and emetics are advisable
b. Acid ingestions - milk or alkali
c. Alkali ingestion - dilute vinegar or citrus fruit juices
d. Antibiotics/ steroids necessary
e. A, b, and c are true
4. Secondary peptic ulcers ore
a. Frequently gastric
b. Commonly solitary
c. Equal frequency in moles and females
d. A, b, and c ore true
e. A & care true
5. True of peptic ulcer disease
a. Hemotemesis - common in primary PUD
b. Scority GI bleeding - common presentation in children with secondary .....
c. Melena more in secondary ulceration
d. A & C
e. All of the above
6 . Surgical intervention in PUD is warranted in
a. Perforation
b. Intermittent hemorrhage
c. Obstruction
d. A & C
e. AOTA
7. Pyloric stenosis
a. Usually occurs in 2 - 5 wks old premature female infant
b. Individual hos voracious appetite, yet facts to thrive
c. ( +) pyloric murmur
d. B & care correct
e. AOTA
8. Associated radiological findings include
a. Double bubble sign
b. Step ladder appearance
c. Pneumoperitoneum
d. A & B only
e. AOTA
9 . Theories/ etiology of necrotizing enterocolitis (NEC)
a. Gastrointestinal ischemia
b. Underdevelopment of GI immune protection
c. Infectious agents
d. A & C
e. AOTA
10. Hirschsprung's disease - highly suspected in
a. Failure to pass meconium during 1st 40 hrs of life
b. Progressive abdominal distention
c. Infant refuses feeding
d. Bilious vomitus
e. AOTA
11. True of hepatitis D virus (HOV)
a. Unable to replicate on its own
b. Less pathologic
c. Capable of infection only when activated by HBV
d. A & Conly
e. AOTA
12. Hepatitis B immune globulin (HBIG) for perinatal exposure
a. HBIG 0.5 ml IM w /in 12 hrs. of birth
b. HB vaccine 10 mg IM w /in 7 days of life, repeat at 1 & 6 mos.
c. HB vaccine 0.5 mg IM w /in 7 days of life, repeat at 1 & 6 mos.
d. A, b, or c
e. A & b only
1 3. Chronic active liver disease
a. Large inoculums of infection agents results in chronic infection
b. Individual with chronic HBV infection have qualitative ad quantitative defect producing antibody
to the virus
c. Altered immunologic mechanism does not allow progression from acute to chronic HBV infection
d . A & c are false
e. A, b, & c are true
14. The main pathology in cholestasis is
a. Obstruction to bile flow
b. Hepatocyte injury
c. Errors in bile acid metabolism
d. AOTA
15. The laboratory results confirmatory of cholestasis
a. 20 % elevation of conjugated fraction bilirubin
b. 20 % increased of unconjugated fraction bilirubin
c. (-) bilirubin
d. (+) stercobilinogen
16. Which laboratory procedures would be the best confirmatory test for choledochal cyst
a. Ultrasound
b. CT scan
c. Nuclear
d. Endoscopic retrograde
17. Which laboratory examination/sis/are considered cholestatic marker/s
a. Unconjugated bilirubin
b. Conjugated bilirubin
c. Alkaline phosphatase
d. Transaminase
18. Decreased intraluminal bile salt in cholestatic jaundice result to one of the ff complications
a. Pruritus
b. Xanthomatas
c. Malabsoq~tion
d. Hypercholesterolemia

19. Hepatitis is considered chronic if transaminase remain elevated for


a. 3
b. 6
c. 12
d. 15 months
20. The marker that will diagnose Hepa A infection is
a. Anti-HA V lgM
b. Anti-HAV lgG
c. Anti-HAY lgA
d. Anti-Hae
21 . What marker would be indicative of active infection and highly infectious
a. Hepa Bs Ag
b . Hepa Be
c. Anti-Hepa C lgM
d. Anti- Hepa Bs
22. The marker produce by vaccination
a . Anti-Hepa Be lgM
b. Anti-Hepa Be lgG
c. Anti-Hepa Be
d . Anti-Hepa Bs
23. Which of these virus is considered parasite to Hepa B
a. Hepa A
b. Hepa B
c. Hepa C
d. Hepa D
e. Hepa E
f. Hepa G
24. Put X on the virus that could become chronic, Y on the virus transmitted by enteral route and Z on the virus
transmitted parenterally
f . Route and Z on the virus transmitted pareaterally
g. Hepa A ____ Y
h. HEpa B X ____ Z
i. Hepa C X ____ Z
j. HEpa D X _ _ _ _ Z
k. Hepa E _ _ _ Y
I. Hepa G X
RENAL
1. Renal vein thrombosis is commonly seen in older patients with:
a. Cyanotic heart disease
b. Angiographic contrast agents
c. Nephrotic syndrome
d. Congenital hypercoagulable state
e. AOTA
2. HUS is characterize by the following except:
a. Microagiopathic hemolytic anemia
b. Diffuse microvascular platelet adhesion
c. Acute renal failure
a. Leukocytosis
e. AOTA
3. All of the following are true except:
a . Signs and symptoms of HSP typically appear 1 - 3 wks after URTI
b. Presence of isolated microscopic hematuria carries the worst prognosis
c. HSP nephritis and lgA nephropathy share a common pathologic result
d. Renal manifestation of HSP nephritis occur up to 1 2 wks after the initial presentation
4. Goodpostures disease is characterize by the following except:
a. Pulmonary hemorrhage
b . Decrease C3 level
c. Immunologic process against type IV collagen
d . NOTA
5. The ultrofiltrate contains all the following substance except
a. Glucose
b. Phosphate
c. Albumin
d. Urea
e. Creatinine
6. A normal kidney of a full term newborn weighs
a. 14 g
b. 21 g
c. 24 g
d. 15 g
7. The GFR maybe optimally measured by
a. BUN
b. CREA
c. 24 hour CHON
d. lnulin
8. Glomerular injury maybe result of the following except
a . Immunologic
b. Genetic
c. Environment
d. Coagulation defect
9. False positive result of urinalysis may occur with the following
a. Fever
b. Alkaline urine
c. High intake of ascorbic acid
d. AOTA
10 . The following are the origin of upper urinary tract sources of hematuria except:
a. Glomerulus
b. Pelvocalyceal system
c. Distal convoluted tubule
d. lnterstitium
11. The primary treatment of lgA nephropathy is
a. lmmunosuppressive therapy
b. Ponisllectomy
c. Renal transplant
d. Use of antihypertensive agents
e. AOTA
1 2. The prototype of acute glomerulonephritis is
a. lgA nephropathy
b. RPGN
c. Acute poststreptococcal glomerulonephritis
d. Alports syndrome
e. AOTA
1 3. All of the following are potential complications of AGN except:
a. Heart failure
b. Hyperkalemia
c. Hyperphosphatemia
d. Hypercalcemia
14. Hematuria is poststreptococcal glomerulonephritis usually persists up to
a. 1 - 2 wks
b. 1 - 2 months
c. 1 - 2 years
d. NOTA
15. Postinfectious cause of RPGN associated with granular immune deposits except:
a. Poststreptococcal glomerulonephritis
b. Bacterial endocarditis
c. Shunt nephritis
d. Visceral abscesses
16. Indication for biopsy are the following except:
a. Significant proteinuria
b. Nephritic syndrome in a child older than 1 0 y / o
c. Hypocomplementemia more than 8 wks
d. NOTA
17. Berger nephropathy is characterize by cresent formation, scarring, deposition of immunoglobulins in the
mesangium. The following are included in the mesangial deposition except:
a. lgM
b. lgG
c. lgE
d. C3
e. lgA
1 8. lgA nephropathy is the most common chronic glomerular disease worldwide. All of the following are true
except:
a. lgA nephropathy is an immune mediated disease caused by ....
b. Seen more often in males
c. Progressive disease may develop in about 20 - 30 % of children
d. Massive proteinuria of more than 100 mg/ 24 hr
19. Most common histologic form of nephritic syndrome
a. Focal segmental glomerulosclerosis
b. Membranous nephropathy
c. Minimal change disease
d. Congenital nephritic syndrome
20. All of the following are complication of nephritic syndrome except:
a. Bacteremia
b . Spontaneous bacterial peritonitis
c. Hypocoagulable state
d. Hyperlipidemia
21 . Hemorrhagic cystitis is associated with the following except:
a. Cyclophosphamide
b. Busulfan
c. Thiotepa
d. Amyloidosis
e. Hepadna virus
22. Common causes of gross hematuria are the following except
a. UTI
b. Trauma
c. Tumor
d. Meatal stenosis
e. B &C
f. C & D
23. The following are etiologic agent for PSAGN except;
a. Streptococcal
b. Gram (-) bacteria
c. Fungal rickettsial
d. B & C
e. C& D
f. NOTA
24. The following are true about anaphylactoid purpura except
a. Renal manifestations occur one wk after the initial manifestations
b. Isolated microscopic hematuria alone carries the best prognosis
c. It is a large vessel vasculitis
d. Signs and symptoms typically appear 1 - 21 days after URTI
e. B & D
f. NOTA

CASE 1
A newborn delivered at NICU with an APGAR score of 3 at birth, 5 at.. minutes px was
unstable. Patient had sudden onset of gross hematuria.
1. What is your impression?
2. What is the risk factor of the case presented?
3. (2) Describe the hematologic picture of the disease?
4. (3) G ive at least 3 risk factor for the above case?

CASE 2
A 2 y/o male had sudden onset of weakness and pallor. ... diarrhea
1 . What is your impression
2. (4) What are the other etiologic agent of the case presented?
I
J
D
C
B
G
E
A
F
H
F

T
F

T
D

C
A
6/14/2020 DISEASES OF THE NEWBORN

DISEASES OF THE NEWBORN


* Required

Email address *

Your email

Hemorrhagic Disease of the Newborn is due to deficient vit k dependent 1 point


factors such as

IX,X,VII and II

IX,X,VIII and II

X,IX,II and III

X,IX,II and V

Which of the following statements is true regarding BPD 1 point

Pulmonary immaturity:imbalance of proteases/antiproteases,imbalance of oxidants-


pro-oxidants

Results from injury to alveoli and airways due to barotrauma and oxygen toxicity

Stiffer compliance,poor recoil,airway hypoactivity

Marked by shifting atelectasis, hyperinflation, increased airway resistance and


decreased work of breathing

https://docs.google.com/forms/d/e/1FAIpQLSePBFRRhPzEYx9fBbR9WogvftzymWVtb-7KuoaBMSu9_4e_Tg/viewform?hr_submission=ChkIppCB6uACEh… 1/23
6/14/2020 DISEASES OF THE NEWBORN

True of clostridium tetani except 1 point

Gr + anaerobe spore forming present in soil and feces

infection can spread through umbilical vessels

symptoms are due to toxin leading to decreased acetyl cholinesterase release

lockjaw is more common in newborns compared to adults.

Ladd’s Procedure is done to correct which of the following: 1 point

Duodenal atresia

Malrotation

Ileal atresia

Pyloric stenosis

Mild acidosis and thrombocytopenia,abdominal wall edema and 1 point

tenderness+/- palpable mass,extensive pneumatosis,early ascitis +/- portal


venous gas.

Bell’s stage II A

Bell’s Stage II B

Bell’s stage III A

Bell’s stage III B

https://docs.google.com/forms/d/e/1FAIpQLSePBFRRhPzEYx9fBbR9WogvftzymWVtb-7KuoaBMSu9_4e_Tg/viewform?hr_submission=ChkIppCB6uACEh… 2/23
6/14/2020 DISEASES OF THE NEWBORN

Risk factors for pneumothorax except: 1 point

undergone intubation

difficult delivery

extubation

Resuscitation

String sign on Barium enema 1 point

Malrotation

GER

Duodenal Atresia

Pyloric stenosis

Included in the Management of PPHN are following except: 1 point

hydration

Oxygenation

Acidification

Sedation

https://docs.google.com/forms/d/e/1FAIpQLSePBFRRhPzEYx9fBbR9WogvftzymWVtb-7KuoaBMSu9_4e_Tg/viewform?hr_submission=ChkIppCB6uACEh… 3/23
6/14/2020 DISEASES OF THE NEWBORN

Involution of right umbilical vein creates a potential weak spot at the 1 point

junction of the right aspect of the umbilical ring and the abdominal wall

Omphalocele

gastroschisis

both

none of the above

Most common radiologic finding in PPHN with idiopathic etiology 1 point

patchy infiltrates

airleaks

white lung

black lung

Highest form of ventilatory support: 1 point

ECMO

High Frequency Ventilation

CPAP

IMV

https://docs.google.com/forms/d/e/1FAIpQLSePBFRRhPzEYx9fBbR9WogvftzymWVtb-7KuoaBMSu9_4e_Tg/viewform?hr_submission=ChkIppCB6uACEh… 4/23
6/14/2020 DISEASES OF THE NEWBORN

True of Turner’s syndrome except 1 point

short neck

short stature

narrow chest

ovarian dysgenesis

Most common solid tumor in the neonatal period 1 point

pheochromocytoma

neuroblastoma

wilm’s tumor

teratoma

Risk factors for GER except: 1 point

Short LES

High LES pressure

Prolonged LES relaxation

Poor esophageal coordination of motility

Clinical presentations of Hirschsprung’s diseases except: 1 point

no meconium in 72 hours

abdominal distension

bloody diarrhea

Constipation or foul smelling liquid stool seepage

https://docs.google.com/forms/d/e/1FAIpQLSePBFRRhPzEYx9fBbR9WogvftzymWVtb-7KuoaBMSu9_4e_Tg/viewform?hr_submission=ChkIppCB6uACEh… 5/23
6/14/2020 DISEASES OF THE NEWBORN

Happy puppet syndrome 1 point

downs syndrome

angelman syndrome

Edward syndrome

Patau syndrome

Has an increased risk of developing Necrotizing Enterocolitis. 1 point

gastroschisis

omphalocele

abdominal hernia

pentalogy of cantrel

Which is true of CDH 1 point

Herniation of intestines through diaphragm secondary to failure of closure of


pleuroperitoneal canal at 10 weeks gestation

Herniation of intestines through diaphragm secondary to failure of closure of


pleuroperitoneal canal at 9 weeks gestation

Herniation of intestines through diaphragm secondary to failure of closure of


pleuroperitoneal canal at 8 weeks gestation

Herniation of intestines through diaphragm secondary to failure of closure of


pleuroperitoneal canal at 7 weeks gestation

https://docs.google.com/forms/d/e/1FAIpQLSePBFRRhPzEYx9fBbR9WogvftzymWVtb-7KuoaBMSu9_4e_Tg/viewform?hr_submission=ChkIppCB6uACEh… 6/23
6/14/2020 DISEASES OF THE NEWBORN

Unusual forces on normal tissue 1 point

dysplasia

disruption

deformation

malformation

Single primary anomaly or mechanical factor initiates a series of events 1 point

that lead to multiple anomalies of the same or separated organ systems


and or body areas

syndrome

sequence

association

complex

The characteristics of pentalogy of cantrel, except 1 point

epigastric abdominal wall defect

upper sternal defect

defect in the anterior diaphragm

defect in the epicardium

intracardiac defects

Option 5

https://docs.google.com/forms/d/e/1FAIpQLSePBFRRhPzEYx9fBbR9WogvftzymWVtb-7KuoaBMSu9_4e_Tg/viewform?hr_submission=ChkIppCB6uACEh… 7/23
6/14/2020 DISEASES OF THE NEWBORN

True of Acute Biliruibin Encephalopathy except 1 point

Yellow staining of the basal ganglia,cranial nerve nuclei and hippocampus

Staining correlates microscopically with necrosis, neuronal loss and gliosis

Hypertonia,lethargy,poorfeeding,poor suck

acute effect of severe hyperbilirubinemia

Tracheo-esophageal fistula can be associated with the following except 1 point

cardiac abnormalities

vertebral defects

radial abnormalities

renal anomalies

none of the above

Breakdown of normal tissue exemplified by amnion rupture 1 point

malformation

disruption

deformation

dysplasia

https://docs.google.com/forms/d/e/1FAIpQLSePBFRRhPzEYx9fBbR9WogvftzymWVtb-7KuoaBMSu9_4e_Tg/viewform?hr_submission=ChkIppCB6uACEh… 8/23
6/14/2020 DISEASES OF THE NEWBORN

Blood beneath the scalp can dissect through subcutaneous tissue of the 1 point

neck and behind the ear; up to 30% of blood volume can be sequestered

cephalhematoma

caput succedaneum

subgaleal hemorrhage

intraventricular hemorrhage

STUDENT NUMBER (12-12345) *

Your answer

Supportive therapy for RDS except: 1 point

thermoregulation

Fluids and nutrition

Circulatory support

oxygen support

Most common congenital defect among babies with Down syndrome 1 point

CAVSD

ASD

PDA

TOF

https://docs.google.com/forms/d/e/1FAIpQLSePBFRRhPzEYx9fBbR9WogvftzymWVtb-7KuoaBMSu9_4e_Tg/viewform?hr_submission=ChkIppCB6uACEh… 9/23
6/14/2020 DISEASES OF THE NEWBORN

LAST NAME (IN CAPITAL LETTERS) *

Your answer

At 34 weeks, the premature has generally attained coordinated 1 point

suck-swallow and defecation reflex

suck-swallow and crying reflex

suck-swallow and breathing reflex

suck-swallow and irritability with moro reflex

Set of morph defects that share a common or adjacent region during 1 point

embryogenesis

syndrome

sequence

association

complex

True of PDA except 1 point

functional closure-48 hrs(FT),in PT,may not have functional closuredays to weeks,can


re open until anatomic closure occurs

Anatomic closure-2-4 weeks of age

becomes ligamentum venosum

all are correct

https://docs.google.com/forms/d/e/1FAIpQLSePBFRRhPzEYx9fBbR9WogvftzymWVtb-7KuoaBMSu9_4e_Tg/viewform?hr_submission=ChkIppCB6uACE… 10/23
6/14/2020 DISEASES OF THE NEWBORN

Which is not true of kernicterus 1 point

hypertonia

extrapyramidal disturbances

hearing loss

mild intellectual deficits

A rare syndrome that causes defects involving the diaphragm, abdominal 1 point

wall, pericardium, heart and lower sternum.

gastroschisis

omphalocele

abdominal hernia

pentalogy of cantrel

Diseases detected by the current Philippine newborn screening are the 1 point

following except

G6PD deficiency

Congenital adrenal hypolasia

galactosemia

Congenital hypothyroidism

https://docs.google.com/forms/d/e/1FAIpQLSePBFRRhPzEYx9fBbR9WogvftzymWVtb-7KuoaBMSu9_4e_Tg/viewform?hr_submission=ChkIppCB6uACEh… 11/23
6/14/2020 DISEASES OF THE NEWBORN

2 or more congenital anomalies occur together more often than expected 1 point
by chance alone and have no well –defined etiology

sequence

association

complex

syndrome

Early onset neonatal sepsis except 1 point

multisystem involvement

lesser mortality compared to late onset sepsis

occurs less than 4-7 days

organism from maternal genital tract

Most common autosomal trisomy 1 point

Down syndrome

Edward syndrome

Patau syndrome

Klinefelter syndrome

https://docs.google.com/forms/d/e/1FAIpQLSePBFRRhPzEYx9fBbR9WogvftzymWVtb-7KuoaBMSu9_4e_Tg/viewform?hr_submission=ChkIppCB6uACE… 12/23
6/14/2020 DISEASES OF THE NEWBORN

The most common cause of anemia among very low birth weight infants is 1 point

early cord clamping during delivery

low erythropoietin stores among the VLBW infants

frequent blood extractions

dilutional anemia due to rapid tissue growth among VLBW infants

Amniotic sac and peritoneum protect the intestinal loops but covering sac 1 point

maybe ruptured.associated defects are present in as many as 80% of


cases.

gastroschisis

omphalocele

abdominal hernia

pentalogy of cantrel

Contraindication/s of phototherapy 1 point

a. bronze baby syndrome

b. high indirect bilirubin level

c. a and b are correct

d. both a and b are emergency indications for photo treatment.

https://docs.google.com/forms/d/e/1FAIpQLSePBFRRhPzEYx9fBbR9WogvftzymWVtb-7KuoaBMSu9_4e_Tg/viewform?hr_submission=ChkIppCB6uACE… 13/23
6/14/2020 DISEASES OF THE NEWBORN

No meconium in first 24 hours,abdominal distention,constipation or liquid 1 point

stool seepage,bloody diarrhea,failure to thrive,urinary obstruction because


of mechanical obstruction by large colon.

intussusception

meconium ileus

hirschprungs disease

ladds bands

Non billous vomiting with dehydration, hypochloremic, hypokalemic 1 point

metabolic alkalosios with string sign on barium enema

doudenal atresia

pyloric stenosis

malrotation

intestinal stricture

The pathophysiology/pathophysiologies of PPHN 1 point

pulmonary bronchocostriction

pulmonary vasodilation

pulmonary vasoconstriction

all of the above

https://docs.google.com/forms/d/e/1FAIpQLSePBFRRhPzEYx9fBbR9WogvftzymWVtb-7KuoaBMSu9_4e_Tg/viewform?hr_submission=ChkIppCB6uACE… 14/23
6/14/2020 DISEASES OF THE NEWBORN

Most common causative micro organisms in early onset neonatal sepsis 1 point

except

GBS

E. coli

L. monocytogenes

acinetobacter baumanii

True of HMD except 1 point

progresses and peaks over 24 to 72 hours

improvement is heralded by diuresis

there is diffuse alveolar atelectasis with severe V/Q mismatch and decreased work of
breathing

it is due to deficiency of surfactant

The following can cause hyperbilirubinemia except 1 point

cephalhematoma

caput succedaneum

subgaleal hemorrhage

all are possible causes of hyperbilirubinemia that is why monitoring should be done
for hyperbilirubinemia among these patients

https://docs.google.com/forms/d/e/1FAIpQLSePBFRRhPzEYx9fBbR9WogvftzymWVtb-7KuoaBMSu9_4e_Tg/viewform?hr_submission=ChkIppCB6uACE… 15/23
6/14/2020 DISEASES OF THE NEWBORN

There is sudden or gradual deterioration of vital signs with chest 1 point

asymmetry and shifting of the cardiac point of maximal impulse

atelectasis

diaphragmatic hernia

pneumothorax

CCAM

True of Klinefelter’s syndrome except 1 point

shyness

immaturity

insecurity

non- assertive activity

True of anemia of prematurity except 1 point

resolves of 3-6 months

can be due to shorter life span of RBCs

blood picture shows normochromic microcytic cells

most common cause is phlebotomy.

https://docs.google.com/forms/d/e/1FAIpQLSePBFRRhPzEYx9fBbR9WogvftzymWVtb-7KuoaBMSu9_4e_Tg/viewform?hr_submission=ChkIppCB6uACE… 16/23
6/14/2020 DISEASES OF THE NEWBORN

Blueberry muffin spots 1 point

Teratoma

neuroblastoma

Wilm’s tumor

hepatoblastoma

Effects of meconium in the lungs are the following 1 point

parenchymal infection

surfactant reactivation

Ball valve effect

all of the above

Isomer changes from 4Z,15Z to 4Z 15E of bilirubin 1 point

structural isomerization

configurational isomerization

chemical isomerization

mechanical isomerization

In PPHN, 2D Echo is done to: 1 point

diagnose PPHN

rule out Congenital cyanotic Heart disease

diagnose Congenital cyanotic heart disease

All of the above

https://docs.google.com/forms/d/e/1FAIpQLSePBFRRhPzEYx9fBbR9WogvftzymWVtb-7KuoaBMSu9_4e_Tg/viewform?hr_submission=ChkIppCB6uACE… 17/23
6/14/2020 DISEASES OF THE NEWBORN

Evicerated intestinal loops without a covering sac though an abdominal 1 point

wall defect located on the right side of the umbilical cord.

gastroschisis

omphalocele

both

pentalogy of cantrel

True of TTN except 1 point

generally resolves by 48 to 72 hours

it is due to delayed resorption of lung fluid leading to alveolar trapping and increased
lung compliance

risk factors include maternal sedation, precipitous delivery and and delivery by C
section

none of the above is incorrect

Associated with Beckwith-weideman syndrome 1 point

teratoma

neuroblastoma

Wilm’s tumor

congenital nephroblastoma

https://docs.google.com/forms/d/e/1FAIpQLSePBFRRhPzEYx9fBbR9WogvftzymWVtb-7KuoaBMSu9_4e_Tg/viewform?hr_submission=ChkIppCB6uACE… 18/23
6/14/2020 DISEASES OF THE NEWBORN

Abnormal organization of cells in tissue 1 point

deformation

malformation

disruption

dysplasia

SECTION *

Choose

True of pathologic jaundice except 1 point

onset before 24 hours

rate of increase > 0.5 mg/dl/hr

there is evidence of ongoing illness

persists > 8 days for preterm and 14 days for term

Associated with cystic fibrosis 1 point

Duodenal atresia

pyloric stenosis

Meconium ileus

Ileal atresia

https://docs.google.com/forms/d/e/1FAIpQLSePBFRRhPzEYx9fBbR9WogvftzymWVtb-7KuoaBMSu9_4e_Tg/viewform?hr_submission=ChkIppCB6uACE… 19/23
6/14/2020 DISEASES OF THE NEWBORN

Viral causes of neonatal pneumonia except: 1 point

HIV

Varicella

CMV

Zika virus

All of the following present with bilious vomiting except 1 point

malrotation

Ileal atresia

Pyloric stenosis

Duodenal atresia

Which has better prognosis 1 point

Omphalocele

gastroschisis

FIRST NAME (IN CAPITAL LETTERS)) *

Your answer

https://docs.google.com/forms/d/e/1FAIpQLSePBFRRhPzEYx9fBbR9WogvftzymWVtb-7KuoaBMSu9_4e_Tg/viewform?hr_submission=ChkIppCB6uACE… 20/23
6/14/2020 DISEASES OF THE NEWBORN

Effects of polycythemia except 1 point

tachypnea

hyperglycemia

hypoxia

ischemia

There is increased enterohepatic circulation among the following except 1 point

intestinal atresia

G6PD

meconium ileus

Hirschsprung’s dse

Only selective vasodilator used in the treatment of PPHN 1 point

magnesium sulphate

iloprost

sildenafil

nitric oxide

Most common bacterial etiology of neonatal pneumonia 1 point

klebsiella and pseudomonas

staphylococci and acinetebacter

enterobacter and citrobacter spp

GBS and E.coli

https://docs.google.com/forms/d/e/1FAIpQLSePBFRRhPzEYx9fBbR9WogvftzymWVtb-7KuoaBMSu9_4e_Tg/viewform?hr_submission=ChkIppCB6uACE… 21/23
6/14/2020 DISEASES OF THE NEWBORN

Increased platelet destruction except 1 point

DIC

Kassaback-merrit

Pregnancy induced hypertension

thombitis

Clinical presentations of polycythemia are the following except 1 point

ischemia

hyperoxia due to increased RBC’s who are very good o2 transporters

tachypnea

hypoglycemia

Hemorrhagic edema, crossing suture lines resolving spontaneously over 1 point

several days

cephalhematoma

caput succedaneum

subgaleal hemorrhage

none of the above.

https://docs.google.com/forms/d/e/1FAIpQLSePBFRRhPzEYx9fBbR9WogvftzymWVtb-7KuoaBMSu9_4e_Tg/viewform?hr_submission=ChkIppCB6uACE… 22/23
6/14/2020 DISEASES OF THE NEWBORN

Sequelae of bilirubin encephalopathy presenting as hypotonia,long term 1 point

neuronal injury, extrapyramidal disturbamnces,hearing loss,gaze


palsies,dental dysplasia and intellectual deficits.

acute bilirubin encephalopathy

cerebral palsy

kernicterus

neuronal plasticity

Most common Congenital Heart problem among the premature 1 point

VSD

ASD

AVSD

PDA

Submit

Never submit passwords through Google Forms.

This content is neither created nor endorsed by Google. Report Abuse - Terms of Service - Privacy Policy

 Forms

https://docs.google.com/forms/d/e/1FAIpQLSePBFRRhPzEYx9fBbR9WogvftzymWVtb-7KuoaBMSu9_4e_Tg/viewform?hr_submission=ChkIppCB6uACE… 23/23
NEONATAL‌‌RESUSCITATION‌  ‌
1. About‌‌___‌‌%‌‌of‌‌newborns‌‌will‌‌require‌‌some‌‌assistance‌‌to‌‌begin‌‌regular‌‌breathing‌‌‌10%‌  ‌
2. About‌‌___%‌‌of‌‌newborns‌‌will‌‌require‌‌extensive‌‌resuscitation‌‌to‌‌survive‌1 ‌ %‌  ‌
3. Careful‌  ‌identification‌‌   of‌‌  risk‌‌  factors‌‌   during‌‌   pregnancy‌‌   and‌‌   labor‌‌  can‌‌   identify‌‌   all‌‌
  babies‌‌   who‌‌   will‌‌ 
require‌‌resuscitation‌‌‌FALSE‌  ‌
4. Chest‌  ‌compressions‌  ‌and‌  ‌medications‌  ‌are‌  ‌(rarely)‌  ‌(frequently)‌  ‌needed‌  ‌when‌  ‌resuscitating‌‌ 
newborns‌  ‌
5. Before‌‌birth,‌‌the‌‌alveoli‌‌in‌‌a‌‌baby’s‌‌lungs‌‌are‌‌(collapsed)‌(‌ expanded)‌‌‌and‌‌filled‌‌with‌(‌ fluid)‌‌‌(air)‌  ‌
6. The‌‌air‌‌that‌‌fills‌‌the‌‌baby’s‌‌alveoli‌‌during‌‌normal‌‌transition‌‌contains‌‌___%‌‌oxygen‌2 ‌ 1%‌  ‌
7. The‌‌   air‌‌  in‌‌
  baby’s‌‌   lungs‌‌   causes‌‌   the‌‌
  pulmonary‌‌   arterioles‌‌   to‌‌  ‌(relax)‌‌
  (constrict)‌‌   do‌‌  that‌‌   the‌‌ oxygen‌‌ 
can‌‌be‌‌absorbed‌‌from‌‌the‌‌alveoli‌‌and‌‌distributed‌‌to‌‌all‌‌organs‌  ‌
8. If‌  ‌a ‌ ‌baby‌  ‌does‌  ‌not‌  ‌begin‌  ‌breathing‌  ‌in‌  ‌response‌  ‌to‌  ‌stimulation,‌  ‌you‌  ‌should‌  ‌assume‌  ‌she‌  ‌is‌  ‌in‌‌ 
___apnea‌‌and‌‌you‌‌should‌‌provide‌‌___‌S‌ ECONDARY;‌‌POSITIVE-PRESSURE‌‌VENTILATIONA‌  ‌
9. If‌  ‌a ‌ ‌baby‌  ‌enters‌  ‌the‌  ‌stage‌  ‌of‌  ‌secondary‌  ‌apnea,‌  ‌her‌  ‌heart‌  ‌rate‌  ‌will‌  ‌(rise)‌  ‌(fall),‌  ‌and‌  ‌her‌  ‌blood‌‌ 
pressure‌‌will‌‌(rise)‌(‌ fall)‌  ‌
10. Restoration‌‌   of‌‌
  adequate‌‌   ventilation‌‌   usually‌‌   will‌‌
  result‌‌  in‌‌
  a ‌‌‌(rapid)‌‌  (gradual)‌‌   (slow)‌‌ improvement‌‌ 
in‌‌heart‌‌rate‌  ‌
11. Resuscitation‌‌(should)‌(‌ should‌‌not)‌‌‌be‌‌delayed‌‌until‌‌the‌‌1-minute‌‌Apgar‌‌score‌‌is‌‌available‌  ‌
12. Premature‌‌babies‌‌may‌‌present‌‌unique‌‌challenges‌‌during‌‌resuscitation‌‌because‌‌of:‌A ‌ OTA‌  ‌
13. Complete‌‌the‌‌missing‌‌parts‌‌of‌‌the‌‌chart‌  ‌
a. Apnea‌‌or‌‌heart‌‌rate‌‌below‌‌___‌1 ‌ 00bpm‌  ‌
b. Provide‌‌___‌‌and‌‌apply‌‌___‌P ‌ OSITIVE-PRESSURE‌‌VENTILATION;‌‌OXIMETER‌‌MONITOR‌  ‌
c. Heart‌‌rate‌‌below‌‌___‌6 ‌ 0bpm‌  ‌
d. Heart‌‌rate‌‌below‌‌___‌6 ‌ 0bpm‌  ‌
e. Insert‌‌an‌‌___‌‌and‌‌give‌‌___‌I‌NTRAVENOUS‌‌LINE‌‌(UMBILICAL‌‌CATHETER);‌‌EPINEPHRINE‌‌IV‌  ‌
14. Every‌‌   delivery‌‌   should‌‌   be‌‌   attended‌‌   by‌‌  at‌‌
  least‌‌
  ___‌‌  skilled‌‌   person‌‌   (s)‌‌  whose‌‌   only‌‌   responsibility‌‌   is‌‌ 
the‌‌management‌‌of‌‌the‌‌newborn‌1 ‌  ‌ ‌
15. If‌  ‌a ‌ ‌high-risk‌  ‌delivery‌  ‌is‌  ‌anticipated,‌  ‌at‌  ‌least‌  ‌___‌  ‌skilled‌  ‌person‌  ‌whose‌  ‌only‌  ‌responsibility‌  ‌is‌‌ 
resuscitation‌‌and‌‌the‌‌management‌‌of‌‌the‌‌baby‌‌should‌‌be‌‌present‌‌at‌‌the‌‌delivery‌2 ‌  ‌ ‌
16. When‌‌   a ‌‌depressed‌‌ newborn‌‌ is‌‌ anticipated‌‌ at‌‌ a ‌‌delivery‌‌ resuscitation‌‌ equipment‌‌ ‌(should)‌‌ (should‌‌ 
not)‌‌be‌‌unpacked‌‌and‌‌ready‌‌for‌‌use‌  ‌
17. A‌‌  baby‌‌   who‌‌   was‌‌  meconium‌‌   stained‌‌   and‌‌ not‌‌ vigorous‌‌ at‌‌ birth‌‌ had‌‌ meconium‌‌ suctioned‌‌ from‌‌ the‌‌ 
trachea‌‌   and‌‌   continued‌‌   to‌‌   require‌‌   supplemental‌‌   oxygen‌‌ to‌‌ keep‌‌ saturation‌‌ as‌‌ measured‌‌ by‌‌ pulse‌‌ 
oximetry‌  ‌(SPO2)‌  ‌>85%.‌  ‌As‌  ‌soon‌  ‌as‌  ‌the‌  ‌heart‌  ‌rate‌  ‌is‌  ‌above‌  ‌100bpm,‌  ‌this‌  ‌baby‌  ‌should‌  ‌receive‌‌ 
(routine)‌(‌ post‌‌resuscitation)‌‌‌care‌  ‌
18. When‌  ‌twins‌  ‌are‌  ‌expected,‌  ‌there‌  ‌should‌  ‌be‌  ‌___‌  ‌people‌  ‌present‌  ‌on‌  ‌the‌  ‌delivery‌  ‌room‌  ‌to‌‌   form‌‌ 
resuscitation‌‌team‌2 ‌ ‌‌FOR‌‌EACH‌‌BABY‌  ‌

   ‌
LESSON‌‌2 ‌ ‌
1. A‌  ‌newborn‌  ‌who‌  ‌is‌  ‌born‌  ‌at‌  ‌term,‌  ‌has‌  ‌no‌  ‌meconium‌  ‌in‌  ‌the‌  ‌amniotic‌  ‌fluid‌  ‌or‌  ‌on‌  ‌the‌  ‌skin,‌  ‌is‌‌ 
breathing‌‌well,‌‌and‌‌has‌‌a‌‌good‌‌muscle‌‌tone‌‌(does)‌(‌ does‌‌not)‌‌‌need‌‌resuscitation‌  ‌
2. A‌‌   newborn‌‌ with‌‌ meconium‌‌ in‌‌ the‌‌ amniotic‌‌ fluid‌‌ who‌‌ is‌‌ not‌‌ vigorous‌‌ ‌(will)‌‌ (will‌‌ not)‌‌ need‌‌ to‌‌ have‌‌ 
his‌‌   trachea‌‌   suctioned‌‌   via‌‌  an‌‌
  endotracheal‌‌ tube.‌‌ A ‌‌newborn‌‌ with‌‌ meconium‌‌ in‌‌ the‌‌ amniotic‌‌ fluid‌‌ 
who‌‌is‌‌vigorous‌‌(will)‌(‌ will‌‌not)‌‌‌need‌‌to‌‌have‌‌his‌‌trachea‌‌suctioned‌‌via‌‌an‌‌endotracheal‌‌tube‌  ‌
3. When‌‌   deciding‌‌   which‌‌   babies‌‌   need‌‌   tracheal‌‌   suctioning,‌‌   the‌‌ term‌‌ “vigorous”‌‌ is‌‌ defined‌‌ by‌‌ what‌‌ 3 ‌‌
characteristics:‌  ‌STRONG‌  ‌RESPIRATORY‌  ‌EFFORTS,‌  ‌GOOD‌  ‌MUSCLE‌  ‌TONE,‌  ‌HR‌  ‌GREATER‌  ‌THAN‌‌ 
100BPM‌  ‌
4. When‌  ‌a ‌ ‌suction‌  ‌catheter‌  ‌is‌  ‌used‌  ‌to‌  ‌clear‌  ‌the‌  ‌oropharynx‌  ‌of‌  ‌meconium‌  ‌before‌  ‌inserting‌  ‌an‌‌ 
endotracheal‌‌tube,‌‌the‌‌appropriate‌‌size‌‌is‌‌___F‌‌or‌‌___F‌1 ‌ 2F‌‌or‌‌14F‌  ‌
5. Which‌  ‌drawing‌  ‌shows‌  ‌the‌  ‌correct‌  ‌way‌  ‌to‌  ‌position‌  ‌a ‌ ‌newborn’s‌  ‌head‌  ‌prior‌  ‌to‌  ‌suctioning‌  ‌the‌‌ 
airway?‌A ‌ .‌‌“SNIFFING‌‌POSITION”‌  ‌
6. A‌  ‌newborn‌  ‌is‌  ‌covered‌  ‌with‌  ‌meconium,‌  ‌is‌  ‌breathing‌  ‌well,‌‌   has‌‌   normal‌‌   muscle‌‌   tone,‌‌   has‌‌  a ‌‌heart‌‌ 
rate‌  ‌of‌  ‌120bpm,‌‌   and‌‌   is‌‌
  pink.‌‌   The‌‌  correct‌‌   action‌‌   is‌‌
  to‌‌
  ‌SUCTION‌‌   THE‌‌  MOUTH‌‌   AND‌‌   NOSE‌‌   WITH‌‌   A ‌‌
BULB‌‌SYRINGE‌‌OR‌‌SUCTION‌‌CATHETER‌  ‌
7. In‌  ‌suctioning‌  ‌a ‌ ‌baby’s‌  ‌nose‌  ‌and‌  ‌mouth,‌  ‌the‌  ‌rule‌  ‌is‌  ‌to‌  ‌first‌  ‌suction‌  ‌the‌  ‌___‌  ‌and‌  ‌then‌  ‌the‌  ‌___‌‌ 
MOUTH;‌‌NOSE‌  ‌
8. Make‌‌   a ‌‌check‌‌   mark‌‌ next‌‌ to‌‌ the‌‌ correct‌‌ ways‌‌ to‌‌ stimulate‌‌ a ‌‌newborn‌‌ ‌SLAPPING‌‌ THE‌‌ SOLE‌‌ OF‌‌ THE‌‌ 
FOOT‌‌AND/OR‌‌RUBBING‌‌THE‌‌BACK‌  ‌
9. If‌‌a‌‌baby‌‌is‌‌in‌‌secondary‌‌apnea,‌‌stimulation‌‌of‌‌the‌‌baby‌‌(will)‌(‌ will‌‌not)‌‌‌stimulate‌‌breathing‌  ‌
10. A‌‌  newborn‌‌   is‌‌
  still‌‌
  not‌‌  breathing‌‌   after‌‌   s ‌‌few‌‌   seconds‌‌   of‌‌ stimulation.‌‌ The‌‌ next‌‌ action‌‌ should‌‌ be‌‌ to‌‌ 
administer‌P ‌ OSITIVE‌‌PRESSURE‌‌VENTILATION‌  ‌
11. A‌‌newborn‌‌has‌‌poor‌‌tone,‌‌labored‌‌breathing‌‌and‌‌cyanosis.‌‌Your‌‌initial‌‌steps‌‌are‌‌to‌A ‌ LL‌  ‌
12. Which‌‌drawings‌‌show‌‌the‌‌correct‌‌way‌‌to‌‌give‌‌free-flow‌‌oxygen‌‌to‌‌a‌‌baby?‌A ‌ LL‌  ‌
13. True‌‌or‌‌false:‌  ‌
a. Oximeters‌‌are‌‌devices‌‌that‌‌measure‌‌the‌‌Po2‌‌of‌‌the‌‌blood‌F‌ ALSE‌  ‌
b. In‌  ‌the‌  ‌delivery‌  ‌room,‌  ‌the‌  ‌oximetry‌  ‌probe‌  ‌should‌  ‌always‌  ‌be‌‌   placed‌‌   on‌‌   the‌‌   baby’s‌‌   right‌‌ 
hand‌‌or‌‌wrist‌T‌ RUE‌  ‌
c. Oxygen‌‌saturation‌‌should‌‌be‌‌expected‌‌to‌‌be‌‌>90%‌‌by‌‌2‌‌minutes‌‌of‌‌age‌F‌ ALSE‌  ‌
d. A‌  ‌baby‌  ‌who‌  ‌is‌  ‌cyanotic‌  ‌and‌  ‌apneic‌  ‌should‌  ‌receive‌  ‌free-flow‌  ‌oxygen‌  ‌as‌  ‌the‌  ‌best‌‌ 
treatment‌F‌ ALSE‌  ‌
14. If‌  ‌you‌  ‌need‌  ‌to‌  ‌give‌  ‌supplemental‌  ‌oxygen‌‌   for‌‌   longer‌‌   than‌‌   a ‌‌few‌‌   minutes,‌‌   the‌‌   oxygen‌‌   should‌‌   be‌‌ 
___‌‌and‌‌___‌H ‌ EATED‌‌AND‌‌HUMIDIFIED‌  ‌
15. You‌‌   have‌‌   stimulated‌‌   a ‌‌newborn‌‌   and‌‌   suctioned‌‌   her‌‌  mouth.‌‌   It‌‌
  is‌‌
  now‌‌   30‌‌   seconds‌‌   after‌‌ birth,‌‌ and‌‌ 
she‌  ‌is‌  ‌still‌  ‌apneic‌‌   and‌‌   pale.‌‌   Her‌‌  heart‌‌   rate‌‌   is‌‌
  80bpm.‌‌   Your‌‌  next‌‌   action‌‌   is‌‌  to‌‌
  ‌POSITIVE‌‌   PRESSURE‌‌ 
VENTILATION‌  ‌
16. You‌‌   count‌‌   a ‌‌newborn’s‌‌   heartbeat‌‌   for‌‌   6 ‌‌seconds‌‌   and‌‌  count‌‌   6 ‌‌beats.‌‌   You‌‌   report‌‌   the‌‌   heart‌‌ rate‌‌ as‌‌ 
___‌6 ‌ 0‌‌BPM‌  ‌
17. An‌‌oximeter‌‌will‌‌show‌‌both‌‌SPo2‌‌and‌‌___‌H ‌ EART‌‌RATE‌  ‌

   ‌
LESSON‌‌3 ‌ ‌
1. The‌‌flow‌‌inflating‌‌bag‌w ‌ ill‌‌not‌‌‌work‌‌without‌‌a‌‌compressed‌‌gas‌‌source‌  ‌
2. A‌‌  baby‌‌  is‌‌
  born‌‌  apneic‌‌   and‌‌  cyanotic.‌‌
  You‌‌
  clear‌‌
  her‌‌   airway‌‌
  and‌‌ stimulate‌‌ her.‌‌ Thirty‌‌ seconds‌‌ after‌‌ 
birth,‌‌she‌‌has‌‌not‌‌improved.‌‌The‌‌next‌‌step‌‌is‌‌to:‌b ‌ egin‌‌positive‌‌pressure‌‌ventilation‌  ‌
3. The‌‌   single‌‌   most‌‌  important‌‌ and‌‌ most‌‌ effective‌‌ step‌‌ in‌‌ neonatal‌‌ resuscitation‌‌ is‌: ‌‌ventilation‌‌ in‌‌ the‌‌ 
lungs‌  ‌
4. Mask‌‌of‌‌different‌‌sizes‌D ‌ O‌‌‌need‌‌to‌‌be‌‌delivered‌‌everyday‌  ‌
5. Self-inflating‌‌   bag‌‌  require‌‌   the‌‌  attachment‌‌  of‌‌
  an‌‌
  ‌OXYGEN‌‌   RESERVOIR‌‌   t ‌‌deliver‌‌
  a ‌‌concentration‌‌ of‌‌ 
oxygen‌‌greater‌‌than‌‌approximately‌‌40%‌  ‌
6. T-piece‌‌resuscitation‌W ‌ ILL‌‌NOT‌‌‌work‌‌without‌‌compressed‌‌gas‌‌source‌  ‌
7. Neonatal‌‌ventilation‌‌bags‌‌are‌M ‌ UCH‌‌SMALLER‌‌‌than‌‌adult‌‌ventilation‌‌bags‌  ‌
8. Self-inflating‌‌bags:‌P ‌ OP-OFF‌‌VALVE‌‌and‌‌PRESSURE‌‌GAUGE‌  ‌
9. Flow‌‌inflating‌‌bag:‌P ‌ RESSURE‌‌GAUGE‌  ‌
10. T-piece‌‌resuscitation:‌M ‌ AXIMUM‌‌RELEASE‌‌CONTROL‌‌and‌‌PRESSURE‌‌GAUGE‌  ‌
11. Free‌  ‌flowing‌  ‌oxygen‌  ‌can‌  ‌be‌  ‌delivered‌  ‌reliably‌  ‌with‌  ‌a ‌ ‌FLOW-INFLATING‌  ‌BAG‌  ‌and‌  ‌T-PIECE‌‌ 
RESUSCITATOR‌  ‌
12. Anatomically‌‌shaped‌‌mass‌‌should‌‌be‌‌positioned‌‌with‌‌the‌P ‌ OINTED‌‌END‌‌‌over‌‌the‌‌newborns‌‌nose‌  ‌
13. Squeeze‌‌the‌‌resuscitation‌‌bag‌‌at‌‌a‌‌rate‌‌of‌4 ‌ 0‌‌to‌‌90‌  ‌

 ‌

   ‌
LESSON‌‌4 ‌ ‌
1. A‌  ‌newborn‌  ‌is‌  ‌apneic‌  ‌and‌  ‌bradycardic.‌  ‌Her‌  ‌airway‌  ‌is‌  ‌cleared,‌  ‌and‌  ‌she‌  ‌is‌  ‌stimulated.‌‌   At‌‌
  30‌‌
  sec,‌‌ 
positive-pressure‌  ‌ventilation‌  ‌is‌  ‌begun.‌  ‌At‌  ‌60‌  ‌seconds,‌  ‌her‌  ‌heart‌  ‌rate‌  ‌is‌  ‌80‌  ‌bpm/‌  ‌chest‌‌ 
compression‌‌   (should)‌‌   ‌(should‌‌   not)‌‌   be‌‌   started.‌‌ Positive‌‌ pressure‌‌ ventilation‌‌ ‌(should)‌‌ ‌(should‌‌ not)‌‌ 
continue‌  ‌
2. A‌‌  newborn‌‌   is‌‌
  apneic‌‌   and‌‌   bradycardic.‌‌   She‌‌   remains‌‌   apneic,‌‌   despite‌‌   having‌‌   airway‌‌   cleared,‌‌ being‌‌ 
stimulated,‌‌   receiving‌‌   30‌‌   seconds‌‌ of‌‌ positive‌‌ pressure‌‌ ventilation‌‌ and‌‌ ensuring‌‌ that‌‌ all‌‌ ventilation‌‌ 
techniques‌‌ are‌‌ optimal.‌‌ Nevertheless,‌‌ her‌‌ heart‌‌ rate‌‌ is‌‌ only‌‌ 40bpm.‌‌ Chest‌‌ compressions‌‌ ‌(should)‌‌ 
(should‌‌not)‌‌be‌‌started.‌‌Positive‌‌pressure‌‌ventilation‌(‌ should)‌‌‌(should‌‌not)‌‌continue‌  ‌
3. The‌‌   heart‌‌   rate‌‌   is‌‌
  40‌‌  bpm‌‌   as‌‌   determined‌‌ by‌‌ auscultation,‌‌ and‌‌ the‌‌ oximeter‌‌ has‌‌ stopped‌‌ working.‌‌ 
Chest‌‌   compressions‌‌   have‌‌   begun,‌‌   but‌‌   the‌‌   baby‌‌   is‌‌
  still‌‌ receiving‌‌ room‌‌ air‌‌ oxygen.‌‌ What‌‌ should‌‌ be‌‌ 
done‌‌about‌‌oxygen‌‌delivery?‌‌(continue‌‌room‌‌air)‌(‌ increase‌‌the‌‌oxygen‌‌concentration‌‌to‌‌100%)‌  ‌
4. During‌‌   the‌‌  compression‌‌   phase‌‌   of‌‌
  chest‌‌ compressions,‌‌ the‌‌ sternum‌‌ compresses‌‌ the‌‌ heart,‌‌ which‌‌ 
causes‌‌   blood‌‌   to‌‌ be‌‌ pumped‌‌ from‌‌ the‌‌ heart‌‌ into‌‌ the‌‌ (veins)‌‌ ‌(arteries)‌. ‌‌In‌‌ the‌‌ release‌‌ phase,‌‌ blood‌‌ 
enters‌‌the‌‌heart‌‌into‌(‌ veins)‌‌‌(arteries)‌  ‌
5. Picture‌  ‌
6. The‌‌preferred‌‌method‌‌of‌‌delivering‌‌chest‌‌compressions‌‌is‌‌the‌(‌ thumb)‌‌‌(2‌‌finger)‌‌technique‌  ‌
7. If‌  ‌you‌  ‌anticipate‌  ‌that‌  ‌the‌  ‌baby‌  ‌will‌  ‌need‌  ‌medication‌  ‌by‌  ‌the‌  ‌umbilical‌  ‌route,‌‌   you‌‌   can‌‌  continue‌‌ 
chest‌  ‌compressions‌  ‌b ‌ ‌one‌  ‌of‌  ‌the‌‌   ff‌‌
  actions:‌‌   ‌MOVING‌‌   TO‌‌
  THE‌‌   HEAD‌‌   OF‌‌
  THE‌‌   BED‌‌   TO‌‌   CONTINUE‌‌ 
THE‌‌THUMB‌‌TECHNIQUE‌‌OR‌‌CHANGING‌‌TO‌‌THE‌‌2-FINGER‌‌TECHNIQUE‌  ‌
8. The‌‌   correct‌‌   depth‌‌   of‌‌
  chest‌‌   compressions‌‌   is‌‌
  approximately‌‌   ‌THIRD‌‌  OF‌‌  THE‌‌ ANTERIOR‌‌ POSTERIOR‌‌ 
DIAMETER‌‌OF‌‌CHEST‌  ‌
9. Picture‌  ‌
10. What‌  ‌phrase‌  ‌is‌  ‌used‌  ‌to‌  ‌time‌  ‌and‌  ‌coordinate‌  ‌chest‌  ‌compressions‌  ‌and‌  ‌ventilation‌‌ 
“ONE-AND-TWO-AND-THREE-AND-BREATHE”‌  ‌
11. The‌‌ratio‌‌of‌‌chest‌‌compressions‌‌to‌‌ventilation‌‌is‌‌___‌‌to‌‌___‌3 ‌ :1‌  ‌
12. During‌  ‌the‌  ‌positive‌  ‌pressure‌  ‌ventilation‌  ‌without‌  ‌chest‌  ‌compressions,‌  ‌the‌  ‌rate‌  ‌of‌  ‌breaths‌  ‌per‌‌ 
minute‌‌should‌‌be‌‌___‌‌to‌‌___‌‌breaths‌‌per‌‌min‌4 ‌ 0-60‌‌BPM‌  ‌
13. During‌  ‌positive‌  ‌pressure‌  ‌ventilation‌  ‌with‌  ‌chest‌  ‌compressions,‌  ‌the‌  ‌rate‌  ‌of‌  ‌“events”‌‌   per‌‌  minute‌‌ 
should‌‌be‌‌___‌‌“events”‌‌per‌‌minute‌1 ‌ 20‌  ‌
14. The‌‌count‌‌“one-and-two-and-three-and-breathe-and-“‌‌should‌‌take‌‌about‌‌___‌‌sec‌2 ‌ ‌‌SEC‌  ‌
15. A‌  ‌baby‌  ‌has‌  ‌required‌  ‌ventilation‌  ‌and‌  ‌chest‌  ‌compressions.‌  ‌After‌  ‌30‌  ‌seconds‌  ‌of‌  ‌chest‌‌ 
compressions,‌  ‌you‌  ‌stop‌‌   and‌‌   count‌‌   8 ‌‌heartbeats‌‌   in‌‌
  6 ‌‌seconds.‌‌   The‌‌
  baby’s‌‌   heart‌‌   rate‌‌   is‌‌
  now‌‌  ___‌‌ 
bpm.‌‌You‌‌should‌‌(continue)‌(‌ stop)‌‌‌chest‌‌compressions‌8 ‌ 0‌‌BPM‌  ‌
16. A‌‌
  baby‌‌   has‌‌  required‌‌   chest‌‌   compressions‌‌   and‌‌   is‌‌
  being‌‌ ventilated‌‌ with‌‌ bag‌‌ and‌‌ mask.‌‌ The‌‌ chest‌‌ is‌‌ 
not‌‌   moving‌‌   well.‌‌   You‌‌   stop‌‌   and‌‌   count‌‌   4 ‌‌beats‌‌   in‌‌  6 ‌‌sec.‌‌   the‌‌ baby’s‌‌ heart‌‌ rate‌‌ is‌‌ now‌‌ ___bpm.‌‌ You‌‌ 
may‌  ‌want‌  ‌to‌  ‌consider‌  ‌40‌  ‌BPM;‌  ‌ENDOTRACHEAL‌  ‌INTUBATION,‌  ‌INSERTION‌  ‌OF‌  ‌AN‌  ‌UMBILICAL‌‌ 
CATHETER‌‌AND‌‌ADMINISTRATION‌‌OF‌‌EPI‌  ‌
17. Complete‌‌the‌‌chart‌  ‌

   ‌
LESSON‌‌5 ‌ ‌
1. A‌‌  new‌‌   born‌‌   with‌‌ meconium‌‌ and‌‌ depressed‌‌ respirations‌‌ ‌WILL‌‌ require‌‌ suctioning‌‌ by‌‌ endotracheal‌‌ 
intubation‌‌before‌‌other‌‌measures‌‌started‌ 
2. Endotracheal‌  ‌intubation‌  ‌SHOULD‌  ‌be‌  ‌considered‌  ‌for‌  ‌a ‌ ‌newborn‌  ‌who‌  ‌is‌  ‌not‌  ‌improving‌  ‌despite‌‌ 
good‌‌technique‌  ‌
3. For‌  ‌babies‌  ‌weighing‌  ‌less‌  ‌than‌  ‌1000g‌  ‌the‌  ‌inside‌  ‌diameter‌  ‌of‌  ‌the‌  ‌endotracheal‌  ‌tube‌  ‌should‌  ‌be‌‌ 
2.5mm‌  ‌
4. The‌‌   blade‌‌   of‌‌
  a ‌‌laryngoscope‌‌
  should‌‌   be‌‌
  ‌no.‌‌
  1 ‌‌for‌‌
  term‌‌
  newborns,‌‌   no.‌‌
  0 ‌‌for‌‌  preterm‌‌  newborns,‌‌ 
no.‌‌00‌‌for‌‌extremely‌‌preterm‌‌newborns‌ 
5. Both‌‌right‌‌and‌‌left‌‌people‌‌should‌‌hold‌‌the‌‌laryngoscope‌‌in‌‌their‌L‌ EFT‌‌hand‌  ‌
6. The‌  ‌goal‌  ‌should‌  ‌be‌  ‌to‌  ‌insert‌  ‌an‌  ‌endotracheal‌  ‌tube‌  ‌and‌  ‌connect‌  ‌it‌  ‌to‌  ‌a ‌ ‌resuscitation‌  ‌device‌‌ 
within‌3 ‌ 0‌‌SECONDS‌  ‌
7. If‌  ‌you‌  ‌have‌  ‌not‌  ‌completed‌  ‌endotracheal‌  ‌intubation‌  ‌within‌  ‌30‌  ‌seconds,‌  ‌REMOVE‌  ‌THE‌‌ 
LARYNGOSCOPE,‌‌VENTILATE‌‌WITH‌‌POSITIVE‌‌PRESSURE‌‌VENTILLATION‌‌MASK‌‌THEN‌‌TRY‌‌AGAIN‌  ‌
8. You‌S‌ HOULD‌‌‌wait‌‌until‌‌the‌‌vocal‌‌cords‌‌are‌‌open‌‌to‌‌insert‌‌the‌‌tube‌  ‌
9. You‌  ‌should‌  ‌insert‌  ‌the‌  ‌tube‌  ‌TO‌  ‌THE‌  ‌LEVEL‌  ‌OF‌  ‌THE‌  ‌VOCAL‌  ‌COD‌  ‌GUIDE‌  ‌AND‌  ‌“ ‌ ‌tip‌  ‌to‌  ‌lip‌‌ 
1-2-3-7-8-9”‌  ‌
10. Laryngeal‌‌mask‌‌airway‌‌device‌‌is‌‌too‌‌large‌‌for‌‌an‌‌extremely‌‌low‌‌birthweight‌‌baby.‌F‌ ALSE‌  ‌

 ‌

   ‌
LESSON‌‌6 ‌ ‌
1. Fewer‌  ‌than‌  ‌___%‌  ‌of‌  ‌babies‌  ‌requiring‌  ‌resuscitation‌  ‌will‌  ‌need‌  ‌epinephrine‌  ‌to‌  ‌stimulate‌  ‌their‌‌ 
hearts‌1 ‌ %‌  ‌
2. As‌‌   soon‌‌   as‌‌
  you‌‌   suspect‌‌   that‌‌
  medications‌‌   may‌‌   be‌‌
  needed‌‌ during‌‌ a ‌‌resuscitation,‌‌ one‌‌ member‌‌ of‌‌ 
the‌‌team‌‌should‌‌begin‌‌to‌‌insert‌‌a(n)‌‌___‌‌to‌‌deliver‌‌the‌‌drug‌U ‌ MBILICAL‌‌VENOUS‌‌CATHETER‌  ‌
3. Effective‌‌   ventilation‌‌ and‌‌ coordinated‌‌ chest‌‌ compressions‌‌ have‌‌ been‌‌ performed‌‌ for‌‌ 45-60‌‌ sec‌‌ the‌‌ 
trachea‌‌   has‌‌ been‌‌ intubated,‌‌ and‌‌ the‌‌ baby’s‌‌ heart‌‌ rate‌‌ is‌‌ below‌‌ 60‌‌ bpm.‌‌ You‌‌ should‌‌ now‌‌ give‌‌ ___‌‌ 
while‌‌continuing‌‌chest‌‌compressions‌‌and‌‌___‌E‌ PI;‌‌VENTILATION‌  ‌
4. What‌  ‌is‌  ‌the‌  ‌potential‌  ‌problem‌  ‌with‌  ‌administering‌  ‌epi‌  ‌through‌  ‌and‌  ‌endotracheal‌  ‌tube?‌‌ 
EPINEPHRINE‌‌   IS‌‌
  NOT‌‌  RELIABLY‌‌   ABSORBED‌‌   IN‌‌
  THE‌‌   LUNGS‌‌   WHEN‌‌   GIVEN‌‌
  BY‌‌
  THE‌‌ ENDOTRACHEAL‌‌ 
ROUTE.‌  ‌A ‌ ‌HIGHER‌  ‌DOSE‌  ‌(0.5-1ML/KG)‌  ‌SHOULD‌  ‌BE‌  ‌CONSIDERED‌  ‌IF‌  ‌EPI‌  ‌IS‌  ‌GIVEN‌  ‌VIA‌‌ 
ENDOTRACHEAL‌‌TUBE‌‌WHILE‌‌UMBILICAL‌‌VENOUS‌‌ACCESS‌‌IS‌‌BEING‌‌ESTABLISHED‌  ‌
5. You‌  ‌should‌  ‌follow‌  ‌an‌  ‌iv‌  ‌dose‌  ‌of‌  ‌epi‌  ‌with‌  ‌a ‌ ‌flush‌  ‌of‌  ‌___‌  ‌to‌  ‌ensure‌  ‌that‌  ‌most‌  ‌of‌  ‌the‌  ‌drug‌  ‌is‌‌ 
delivered‌‌to‌‌the‌‌baby‌‌and‌‌not‌‌left‌‌in‌‌the‌‌catheter‌N ‌ ORMAL‌‌SALINE‌  ‌
6. Epi‌  ‌(increases)‌  ‌(decreases)‌  ‌the‌  ‌bp‌  ‌and‌  ‌strength‌  ‌of‌  ‌cardiac‌  ‌contractions‌  ‌and‌  ‌(increases)‌‌ 
(decreases)‌‌the‌‌rate‌‌of‌‌cardiac‌‌contractions‌  ‌
7. The‌‌recommended‌‌concentration‌‌of‌‌epi‌‌for‌‌newborn‌‌is‌‌(1:1000)‌(‌ 1:10000)‌  ‌
8. The‌‌   recommend‌‌ dose‌‌ of‌‌ epi‌‌ for‌‌ newborns‌‌ is‌‌ ___‌‌ to‌‌ ___‌‌ ml/kg,‌‌ if‌‌ given‌‌ iv‌‌ and‌‌ ___‌‌ to‌‌ ___‌‌ ml/kg,‌‌ if‌‌ 
given‌‌endotracheally,‌‌of‌‌a‌‌1:10000‌‌soln‌0 ‌ .1-0.3ML/KG;‌‌0.5-1ML/KG‌  ‌
9. Epi‌‌should‌‌be‌‌given‌‌(slowly)‌(‌ as‌‌quickly‌‌as‌‌possible)‌  ‌
10. What‌‌should‌‌you‌‌do‌‌approximately‌‌1‌‌minute‌‌after‌‌giving‌‌epi?‌C ‌ HECK‌‌THE‌‌HEART‌‌RATE‌  ‌
11. If‌‌
  the‌‌   baby’s‌‌   heart‌‌   rate‌‌
  remains‌‌   below‌‌
  60bpm,‌‌   you‌‌  can‌‌
  repeat‌‌  the‌‌
  dose‌‌
  of‌‌
  epi‌‌ every‌‌ ___‌‌ to‌‌ ___‌‌ 
min‌3 ‌ -5‌‌MIN‌  ‌
12. If‌‌
  the‌‌   baby’s‌‌   heart‌‌   rate‌‌
  remains‌‌   below‌‌ 60bpm‌‌ after‌‌ you‌‌ have‌‌ given‌‌ epi,‌‌ you‌‌ should‌‌ also‌‌ check‌‌ to‌‌ 
make‌‌   sure‌‌   that‌‌   ventilation‌‌ is‌‌ producing‌‌ adequate‌‌ lung‌‌ inflation‌‌ and‌‌ ___‌‌ are‌‌ being‌‌ done‌‌ correctly‌‌ 
CHEST‌‌COMPRESSIONS‌  ‌
13. If‌  ‌the‌  ‌baby‌  ‌appears‌  ‌to‌  ‌be‌  ‌in‌  ‌shock,‌  ‌there‌  ‌is‌  ‌evidence‌  ‌of‌  ‌blood‌  ‌loss‌  ‌and‌  ‌resuscitation‌  ‌is‌  ‌not‌‌ 
resulting‌  ‌in‌  ‌improvement,‌  ‌you‌  ‌should‌  ‌consider‌  ‌giving‌  ‌__ml/kg‌  ‌of‌  ‌___‌  ‌by‌  ‌what‌  ‌route?‌  ‌10;‌‌ 
VOLUME‌‌EXPANDER‌‌BY‌‌UMBILICAL‌‌VEIN‌  ‌

You might also like